.
.
Math Central - mathcentral.uregina.ca
Quandaries & Queries
Q & Q
. .
topic card  

Topic:

algebra

list of
topics
. .
start over

547 items are filed under this topic.
 
Page
1/1
Fibonacci numbers 2021-08-25
From John:
Make a single column of numbers. Start with two numbers of your choice. The third number is the sum of the previous two, the fourth number is the sum of numbers two and three, and so on until you have ten numbers in the column. Add up all ten numbers. Now, take the seventh number and multiply it by eleven. This product will equal the sum of the ten numbers. The same result will occur regardless of the first two numbers chosen. The question is why does the 7th number multiplied by 11 always equal the sum of the ten numbers?
Answered by Penny Nom.
Comparing two job offers 2020-11-17
From Selena:
Raja has been offered two jobs.
Each of these jobs takes 24 weeks to complete.
One job pays $3440 every 8 weeks. The other job pays $2700 every 6 weeks. Raja wants to accept the job that pays more per week.
Show how to use equations to help Raja make her choice.

Answered by Penny Nom.
The difference between two number is 7 2020-01-07
From Jaypercey:
the difference between two number is 7 find the two number if the larger 3times the smaller
Answered by Penny Nom.
Beginning algebra 2019-11-19
From Maya:
Two numbers have a sum of 57. Three times the first number is 13 less than the second number. Find the numbers.
Use let statements to help you solve. (Grade 9 math)
-Maya (help would be appreciated)

Answered by Penny Nom.
The sum of the ages of Ruth and her mother is 77 years. 2019-11-06
From Lydia:
The sum of the ages of Ruth and her mother is 77 years. The difference in their ages is 27 years. How old is each?
Answered by Penny Nom.
Algebra 2019-11-06
From Krizhia:
How is y= 1/2 * (x-1)² be x= √2y + 1. Please show me the steps
Answered by Penny Nom.
Simultaneous equations 2019-10-16
From deepak:
This is too complicated please help me
8/x - 10/y = 1 and x+y=9

Answered by Penny Nom.
The difference of two numbers is 42 2019-10-14
From Patrick:
The difference of two numbers is 42 and the larger number is 8/5 of the smaller number. Find the two numbers.
Answered by Penny Nom.
Four less than three times a number is 20 2019-09-29
From Linda:
four less than three times a number is 20,what is the number?
Answered by Penny Nom.
Can one equation with two variables be solved algebraically? 2019-08-30
From Don:
Can one equation with two variables be solved algebraically?
Answered by Penny Nom.
David Is 6 years older than his sister Delia 2019-08-27
From Abigail:
David Is 6 years older than his sister Delia. In 10 years the sum of There age 52. how old are David and Delia today
Answered by Penny Nom.
Subdividing land 2019-05-09
From Reuben:
This is the measurements of my plot, A-B 46.7M, B-C 193.1, C-D 198.5 & D-A 208.25 (Clockwise naming of sides) angle A at 90 degrees. My questions is how do i subdivide this plot from the bottom having lines running parallel to C-D, eg two 2acre plots. the the remaining part becomes my compound (Uper part at line A-B)
Answered by Harley Weston.
The area of a triangle 2019-04-25
From troy:
The base and the height of a right triangle are represented by the expressions below. The area of the triangle is 28 square units. Find the lengths of the base and the height of the triangle

base= x+4
height=x-6

Answered by Penny Nom.
Weighing stars and hearts 2019-02-25
From Zach:
Two stars and one heart weigh 38 units. One star and one heart weigh 27 units what is the weight of each?
Answered by Penny Nom.
Adding and multiplying two numbers 2019-02-13
From katie:
what 2 numbers add to get 7 and multiply to get 2?
Answered by Penny Nom.
The sum of three consecutive even integers 2019-01-27
From Pauleen:
Translate this into algebraic expression
The sum of three consecutive even integers, if x is the first even integer

Answered by Penny Nom.
Sale price and employee discount 2019-01-18
From Margaret:
What is the formula to find the 75% off full price item that was already marked 30% off?
Answered by Penny Nom.
Solve for x in Y=1.5x + log (1+x) 2018-12-23
From Asadollah:
Please restate below expression in terms of y

Y=1.5x + log (1+x)

Thanks

Answered by Penny Nom.
How many solutions does 3 (x + 4) = 3 x + 4 have? 2018-12-01
From Milan:
how many solutions does 3 (x + 4) = 3 x + 4 have?
Answered by Penny Nom.
Terry was two-thirds as old as Nia 2018-11-19
From Makenzy:
four years ago, terry was two-thirds as old as nia. now terry is three-fourths as old as nia. how old is each now
Answered by Penny Nom.
What was the original number? 2018-11-18
From jaydaleigh:
Mark was thinking of a number. Mark adds 11 to it, then doubles it and gets an answer of 40.5. What was the original number?
Answered by Penny Nom.
The sum and difference of two numbers 2018-09-17
From Feni:
The sum of two numbers is 4. The difference of the two numbers is 2. What are the two numbers?
Answered by Penny Nom.
Robbie and fran run laps 2018-08-26
From khansa:
robbie runs 3 laps for unknown number of days. fran runs 3 times as many laps each day for unknown number of day. what is the formula for their combined total number of laps
Answered by Penny Nom.
A cube in a cylinder 2018-08-13
From Geli:
A cylinder with a radius r and height 2r+4 contains a cube with edge length r√2. What fraction of cylinder volume is taken up by the cube
Answered by Penny Nom.
Creating a number sentence 2018-06-29
From trish:
First I have to solve the problem. Then write a number sentence for the problem. The problem: By installing a $120.00 thermostat that reduces the temperature setting at night a family hopes to cut its annual fuel bill for heating oil by 8%, and recover the cost of the thermostat in fuel savings after 2years. What was the family's annual fuel bill before installing the thermostat? Thanks can't figure this one out!
Answered by Penny Nom.
$96 is 66 2/3% of what dollar amount? 2018-05-31
From mary:
$96 is 66 2/3% of what dollar amount?
Answered by Penny Nom.
Two expressions for the perimeter of a a rectangle 2018-05-13
From Slayde:
A rectangular house has one side 4 metres longer than the other. Create two different expressions for its perimeter. The expressions should be given in factorised and expanded form.
Answered by Penny Nom.
The mass of a durian 2018-04-14
From REEVASSHINIE:
A watermelon with a mass of 5 kg and two durians with the same mass are placed on an electronic balance.If the reading shown on the balance is m kg,write an expression for the mass of a durian
Answered by Penny Nom.
The sum and difference of two numbers 2018-03-13
From samima:
Two numbers have a difference of 0.85 and the sum 1.What are the numbers?
Answered by Penny Nom.
Expand (5x-9)(5x+9) 2018-03-01
From adil:
expand the following:
(5x-9)(5x+9)

Answered by Penny Nom.
y as an expression in terms of x 2018-01-20
From Mark:
What is y as an expression in terms of x ?
Answered by pennt.
xy+x=2y. Solve for y 2018-01-10
From Hugh:
xy+x=2y. Solve for y
Answered by Penny Nom.
Macy and Camile's ages 2017-09-30
From carolyn:
macy is 2 years older than Camille twice the sum of their ages is 52 which equation can be used to find camilles age x
Answered by Penny Nom.
(x-18)/7=6 2017-08-08
From David:
(x-18)/7=6
Answered by Penny Nom.
A police officer spots a speeding car 2017-07-20
From Josh:
Marie, a police officer, spots a speeding car and starts chasing it. The speeding car travels at a speed of 130 feet per second. Marie's car reaches a constant speed of 145 feet per second 1725 feet from the start of the chase. During that time, the speeding car has traveled 3150 feet. Write a system of equations to represent this situation.
Answered by Penny Nom.
Catherine, Molly and Tom 2017-07-19
From liezel:
what is the answer of this problem, If Catherine and molly has 320pesos altogether, Molly and tom have 360 in all. Tom and Catherine have 240pesos altogether.How much does each of them have?
Answered by Penny Nom.
y = g(x + 1) 2017-06-27
From Shamsudiin:
HI there i am currently doing a level maths AS

I don't understand how to graph this Question y=g(x+1) i really need help please please respond to this problem

Answered by Penny Nom.
Algebra 2017-05-28
From Kerim:
I need to solve this equation.
y=at^2 - 2at
x =2a√t

Express y in terms of x and a

Give in the form y=(x^p/ma^3) - (x^q/na) where p,q,m and n are integers

Answered by Penny Nom.
Jamie has twice as many video games as Mel. 2017-04-27
From John:
Jamie has twice as many video games as Mel. Together, the boys have 24 games. how many games do each boy have
Answered by Penny Nom.
Express x/a -2a/x =-1 in terms of a. 2017-04-14
From tony:
express x/a -2a/x =-1 in terms of a.
Answered by Penny Nom.
13 - 3x = x times the square root of 5 2017-03-29
From Kevin:
A student I am tutoring was sent this question from his teacher.

Solve for x: 13 - 3x = x times the square root of 5

Answered by Penny Nom.
A total amount of $27,500 invested at 5% and 7% 2017-02-19
From Brad:
Kristina invests a total of $27,500 in two accounts paying 5% and 7% simple interest, respectively. How much was invested in each account if, after one year, the total interest was $1,645.00.
Answered by Penny Nom.
Five equations 2017-01-16
From Muhammed:
A x 4 = E
B ÷ 4 = E
C + 4 = E
D - 4 = E
___________
& A + B + C + D = 100
what is the value of E

Answered by Penny Nom.
Renting an electric cart 2017-01-07
From Desiree:
Vacation Rentals rents electric carts for travel on their grounds. The economy plan rents carts for $26 per day and the premium plan rents a carts for $12 per day plus a one time fee of $126.05
For how many whole days would it be cheaper to rent from the economy plan then from the premium plan?

Answered by Penny Nom.
Joining a fitness center 2016-12-18
From Salem:
A fitness center offers a special price for group-class students. The cost of registration is 30 dollars and then you have to pay 25 dollars per month. What is the rule of the equation in the light of
c=---m+----?

Answered by Penny Nom.
1/1-cosine(2x) - 1/1+ cos(2x) 2016-12-14
From Sean:
1/1-cosine(2x) - 1/1+ cos(2x)
Answered by Penny Nom.
3^2x-5=27 2016-12-14
From danaja:
solve for the following 3^2x-5=27
Answered by Penny Nom.
A stone walkway around a garden 2016-11-27
From Abigail:
A garden measures 12 feet by 16 feet. The family would like to add a stone walkway around the garden. This would increase the total area of to 302.25 square feet. what will be the width of the of the walkway
Answered by Penny Nom.
An equation with fractions 2016-11-09
From Tina:
How do you solve x+1/3=x+2/5. X+1 divided by 3= x+2 divided by 5. What's x=
Answered by Penny Nom.
Standard form 2016-09-22
From anna:
ok. i have b=2/3x+5/7. im supposed to change it to standard form. how do i do that?
Answered by Penny Nom.
Two equations 2016-09-17
From Nilesh:
How to solve (x-y)^1/2 + 3(x-y) = 30 ; xy + 3(x-y) = 11?
Answered by Penny Nom.
Todd Matt and Jeff 2016-08-30
From Lisa:
Three brothers spent different amounts of money on their recent vacation.
Matt spent $25 less than Todd.
Jeff spent 3 times as much as Matt.
How much did each brother spend if they spent a total of $275?

Check answer required
T= Todd

Answered by Penny Nom.
I bought 3 books for $150.00 2016-08-20
From Shavon:
if i bought 3 books for $150.00 and 1 of the books cost 50% more than both other books combined what's the price of the more expensive book?
Answered by Penny Nom.
Carlos, Mark, Jeremy, and Alan collect Yugioh cards 2016-08-08
From justin:
Carlos, Mark, Jeremy, and Alan all collect Yugioh cards. Mark has twice as many cards as Jeremy. Carlos has 3 times as many cards as Jeremy, and Alan has collected 12 less than 3 times as many cards as Jeremy. Together the four players have collected 456 cards. How many does each person have?
Answered by Penny Nom.
The dimensions of a rectangle 2016-07-25
From Krunox:
A rectangle is twice as long as it is wide. Its perimeter (the sum of the lengths of its sides) is 60 cm. Find its length and width.
Answered by Penny Nom.
The composition of a function with itself 2016-07-17
From Mel:
If f(1 + 3x) = 1 * x, solve f(f(x))
Answered by Penny Nom.
If x+y=7 and x^2+y^2=42, then xy equals? 2016-07-17
From Mehzad:
If x+y=7 and x^2+y^2=42, then xy equals?
Answered by Penny Nom.
A train from Erie to New York 2016-07-14
From carol:
a train leaves Erie with twice as many women as men. At York, 17 men get on and 16 women . How many men and women were originally on the train?off. There are now the same number of men as women
Answered by Penny Nom.
An equation involving pi 2016-06-26
From Diaz:
How would you solve :

( pie*X)/ 4 = -pie/2

I have no idea.what to do!

Answered by Penny Nom.
One million is six percent of what number? 2016-04-26
From Colleen:
One million is six percent of what number?
Thanks!

Answered by Penny Nom.
A square of side length 4x-3 units 2016-04-20
From Bryan:
Find an expression that represents the area of a square with side length 4x-3
Answered by Penny Nom.
Conversion of an equation 2016-04-11
From Marie:
How to you convert the equation of a line from x-intercept form (y=(x-x1)(x-x2)) to general form (y=ax^2+bx+c) if it involves square roots?

For example:
Convert this equation to general form: y=(x+the square root of 5)^2

Thanks for the help!

Answered by Penny Nom.
When does sqrt(x^2+y^2)=x+y? 2016-03-30
From Margret:
when does sqrt(x^2+y^2)=x+y
Answered by Penny Nom.
The perimeter of a rectangle 2016-03-17
From Linda:
Noel correctly adds the length of three sides of a rectangle and gets 88 cm her brother Ryan correctly adds the lengths of three sides of the same rectangle and gets 80cm. What is the number of cm in the perimeter of the rectangle
Answered by Penny Nom.
A word problem 2016-03-10
From Sam:
Jo is 10 cm taller than Ruth. Pam is 16cm taller than Ruth .Rod is 8cm taller than Pam. How much taller than Jo is Rod?
Answered by Penny Nom.
The area of a rectangle 2016-03-03
From Lucas:
The area of a rectangle is given by A=x2+18x+72
-use factoring to find an expression for the dimensions of the rectangle.
-if the area of the rectangle is 7 square feet, what are the possible values of x?
-what are the dimensions of a rectangle?

Answered by Penny Nom.
Shooting a ball at a target 2016-02-16
From Thys:
Hi
I have a problem with the formula that i use .(for programming)
I have looked all over the web to find a solution but no luck.
I have a cannon that shoots a ball at a target
I use this formula to calculate what my initial velocity must be to hit the target at a angle of 30 degrees and a distance of 15m (the cannon and target position is known) It works perfectly if both is at same height but if one is higher or lower it miss.

In an example I am working with the range is 30m, the angle is 45 degrees and the target is 10m higher than my position.
Please help
Formula = V0 = √RG / Sin(2α)

Answered by Harley Weston.
Two equations in two unknowns 2016-02-13
From Anumba:
4x + 2y = 4
7x - y = -11

Answered by Penny Nom.
Todd is 3 years older than his brother Jack 2016-02-13
From Victoria:
Todd is 3 years older than his brother Jack. If Jack is x years old and Todd is y years old, write a rule that relates their ages over time. When Jack is 28 years old, how old will Todd be?
Answered by Penny Nom.
A 2 digit number 2016-02-08
From Lloyd:
The sum of digits of a certain 2 digit number is 7. Reversing its digits increases the number by 9. Find the number
Answered by Penny Nom.
The equation of a vertical line 2016-02-07
From Kagiso:
The equation for a vertical line that goes through the point (7; -3) is?
Answered by Penny Nom.
38 is what percent of 400? 2016-02-07
From chris:
38 is what percent of 400?
Answered by Penny Nom.
When will Scott and Jill have earned the same amount? 2016-02-04
From kalia:
Scott starts with $10 and earns $5 and hour. Jill starts with $40 and earns $3 and hour. when will the two have the same amount of money in the same hours?
Answered by Penny Nom.
Solve 1/3 ( x-5 ) + 4 =1/6 ( 2 - 3 x ) +1 for x 2016-01-15
From edwin:
1/3 ( x-5 ) + 4 =1/6 ( 2 - 3 x ) +1
Answered by Penny Nom.
A rectangular enclosure 2016-01-15
From Amber:
A rancher wants to use 300ft of fencing to enclose a rectangular area of 4400 square feet. What dimensions should the rectangle be?
Answered by Penny Nom.
A triangular shaped garden 2016-01-13
From Demetrie:
Dwayne's garden is triangle-shaped with two equal sides and a third side that is 4 ft more than the length of an equal side. If the perimeter is 49 ft, how long is the longest side?
Answered by Penny Nom.
Four consecutive integers 2016-01-05
From sherica:
my teacher gave us an assignment and it says here that; THERE ARE FOUR CONSECUTIVE INTEGERS.IF THE SMALLEST ONE IS x ,FIND THE SUM OF THE FOUR INTEGERS. that is the question ...thanks hope that u can answer it
Answered by Penny Nom.
Proof that an erroneous algebraic statement is false 2015-12-14
From Berteanu:
I need help with this proposition:
"It exists x a real number that for every y real number 5*x-2*y*y=1
This is false.
Let x be from R.
And I need an y real number that 5*x-2*y*y!=1
Please,could you help me?

Answered by Penny Nom.
I was wondering what xy means. 2015-09-30
From Eric:
I was wondering what xy means. My two things I think it means are the two digits of x and y or x times y. thanks for your time, Eric
Answered by Penny Nom.
The length of a rectangle 2015-09-26
From Tris:
Hi,
I'm trying to figure out the length of a rectangle but i forgot how. So, the width is 5cm and the perimeter is 34cm.

Answered by Penny Nom.
The weight of a pumpkin 2015-09-01
From Lisa:
A pumpkins weighs 10 pounds plus half of 10 pounds plus half it's own weight, what does the pumpkin weigh?
Answered by Penny Nom.
The diameter and circumference of a circle 2015-09-01
From Tracey:
Hello,
I own a custom workroom, I am figuring out fabric quantities or an estimate. When at the job site, I forgot to measure the diameter of the semi circle shape that I have to make a cushion for!!!
If the circumference of the semi circle is 165" what would the diameter be, maybe half this measurement????

I appreciate any help...I basically need the width from left to right to see if I can fit the cushion on one width of fabric that is 52" wide!

Answered by Penny Nom.
Manipulating an equation 2015-06-05
From Leigh:
Hi, I'm having trouble trying to find out how the equation y=mx+b for b comes up with the answer b=y-mx. Could you please assist me with this
Answered by Penny Nom.
A two digit number 2015-06-05
From Madison:
Trini took some photos. The number of photos she took is a two digit number. The sum of the digits is 11. The tens digit is 3 more than the ones digit. How many photos did Trini take?
Answered by Penny Nom.
The perimeter of a rectangle 2015-05-28
From Imran:
The length of a rectangle is x+3 centimetres.
The width of the rectangle is x-1 centimetres.

Find an expression in terms of x for the perimeter of the rectangle.
The perimeter of a rectangle The perimeter of a rectangle
Give your expressions in it's simplest form.
The perimeter of a rectangle The perimeter of a rectangle
Thank You.

Answered by Penny Nom.
A word problem 2015-05-08
From JAMES:
A businessman bought a number of similar articles for a total of sh. 6000. Three of the articles turned out to be defective with no resale value. He sold the remaining articles at 12.5% each above the cost price making a total profit of sh 480. How many articles had he bought?
Answered by Penny Nom.
Expand and Simplify (3x+5)(5x-9) 2015-04-23
From chloe:
Expand and Simplify (3x+5)(5x-9)
Answered by Penny Nom.
The area of a circle given the circumference 2015-04-12
From F:
How do you find the area of a circumference of 6?
Thanks.

Answered by Penny Nom.
Stopping distance 2015-02-11
From Audrey:
A Harley Davidson motor cycle weighing 650 lbs with a rider weighing 175 bs was going 40 miles per hour. He had to make a sudden stop . Slammed on brakes. How long a distance for him to stop?
Answered by Penny Nom.
2x^2-2x+1=4x+1 2015-02-06
From Caitlyn:
2x^2-2x+1=4x+1
Answered by Penny Nom.
Russell is 6 years older than Samantha 2015-01-26
From Kelli:
Russell is 6 years older than Samantha. In 4 years the sum of their ages will be 78. How old is Russell now?
Answered by Penny Nom.
h+h+h = -9 2015-01-12
From juanita:
How to solve h+h+h = -9
Answered by Penny Nom.
Two quadratic equations 2014-11-20
From rajesh:
Sir,
I would like to discuss on question-
-Find the value of variable X and Y, equations are
X2+Y2+2XY=25
X2+Y2-2XY=1

Sir, my point is equation looks not so easy.My opinion is there many (more than 4)value possible for both X and Y.
Kindly help me.
Waiting for your response.
regards,
rajesh

Answered by Penny Nom.
(x-3)^2-(x+3)^2 2014-11-13
From Bernice:
(x-3)^2-(x+3)^2
Answered by Penny Nom.
1/6k=-11 2014-11-13
From Marty:
Using the balance method, what is the answer to 1/6k=-11, trying to find what k equals. Thanks
Answered by Penny Nom.
How does pir^2 = 1/4pid^2? 2014-10-14
From al:
Hi I cant work out the algebra. How does pir^2 = 1/4pid^2 Thanx
Answered by Penny Nom.
15 - (5-x)/ x = 30 2014-10-13
From AMANDA:
15 - (5-x)/ x = 30

I thought you had to multiply both sides by x in order to remove the fraction?

15x - 5x - x2 = 30x

then combine 10x - x2 = 30x

-x2 =20x

then take the sqrt of each side...

what am I doing wrong??

Answered by Penny Nom.
Pre algebra 2014-10-12
From mohamed:
i want to know the meaning of pre algebra
Answered by Penny Nom.
Sum of 22, product of 125 2014-07-03
From marj:
find two real numbers whose sum is 22 and whose product is 125.
Answered by Paul Betts, Robert Dawson and Penny Nom.
If x+y+z=1 then x^2 + y^2 + z^2 = ? 2014-04-30
From Nitin:
If x+y+z=1 then x^2 +y^2+z^2 = ? ?
Answered by Robert Dawson.
The cost before the sales tax 2014-04-13
From Juanda:
Hello,

I know the customer cost with tax and I know the sales tax.
How do I find out the customer cost prior to the added sales tax?

Thank you

Answered by Penny Nom.
Manipulating an equation 2014-04-12
From Paul:
S=4wl+2wh and solve for h
Answered by Penny Nom.
Evaluate x-(3x+5) if x=-2 2014-04-03
From Destiny:
I wanted to know the value of x-(3x+5) if x=-2
Answered by Penny Nom.
Ms. Werenich's nickels, dimes and quarters 2014-03-19
From London:
Ms. Werenich has twice as many dimes as nickels and 4 more quarters than nickels. If she has $4.50, how many of each coin does she have? What would the let statements be of this problem and can you solve it?
Answered by Penny Nom.
Find two numbers with a sum of -8 and a difference of 4 2014-03-19
From Elisabeth:
Find two numbers with a sum of -8 and a difference of 4. Let x be the greater number and y be the lesser number.
Answered by Penny Nom.
A word problem 2014-03-09
From Joanne:
Ashley weighs 5kgs more than Joyce. Their total weight is 41 kgs. What is the weight of Ashley?
Answered by Penny Nom.
A word problem 2014-02-23
From Anh:
The Texas State Legislature is comprised of state senators and state representatives. The sum of the number of senators and representatives is 181. There are 119 more representatives than senators. How many senators and how many representatives make up the Texas Legislature?
Answered by Penny Nom.
x^3+(1/x^3)=18√3 2014-02-23
From Asiv:
if x^3+(1/x^3)=18√3; then what is the value of x?
Answered by Penny Nom.
A fountain surrounded by a walk 2014-02-18
From Maddie:
A square Water Fountain is Surrounded by a walk 2 yards wide. If the Area of the Fountain is 2/3 of the total area of the Fountain and the Walk. What are the Outside Dimensions of the Walk
Answered by Penny Nom.
The graph of y=2x+4 2014-02-18
From tyler:
y=2x+4 graph
Answered by Penny Nom.
Paul is making banana bread 2014-02-17
From John:
Paul is making banana bread. The number of cups of bananas he uses is proportional to the number of loaves of bread. Paul uses 11 1/4 cups of bananas to make 5 loaves of bread. Which equation represents the relationship between c, the number of cups of bananas, and b, the number of loaves of bread?
A) C=4/9 b
B) C=2b
C) C= 2 1/4 b
D) C=5b

Answered by Penny Nom.
The growth of a tree 2014-02-16
From Emily:
If a tree is 10 ft tall after 2 years and 25 ft tall after 5 yrs , find the rate of change of growth Of the tree if y is the height in ft and x is the number of years.
Answered by Penny Nom.
Rolls of window film 2014-02-14
From Travis:
This question is probably close to the same question as "roll of paper"

We have Rolls of Window Film that we are trying to figure out an equation for a spreadsheet that we can use to "inventory" our window film.

We use a caliper tool to measure the thickness of the roll in millimeters.

the core thickness = 1.90mm
Full Roll thickness(including core) = 9.08mm to 9.12mm
Film thickness = 0.06

Full Roll of Film is supposed to average 1200" of film

What equation could we use to get the approximate inches left remaining on the roll if we measured the roll including the core with the Caliper tool in Millimeters?

Answered by Harley Weston.
The dimensions of a room 2014-02-01
From jenna:
The length of a rectangular room is 2m more than the width. Find the dimensions if the perimeter of the room is 34m
Answered by Penny Nom.
Kate, Tess and Nina 2014-01-23
From Sam:
Kate is 16 yrs. younger then Tess. Nina is 8 yrs.older then Kate. The sum of their ages is 189. How old is each?
Answered by Penny Nom.
Reducing the area of a parking lot 2014-01-19
From Robin:
The school is reducing the area of its 20yd by 50yd parking lot by removing a strip at one end of the parking lot and another strip of the same width from one side. The new area of the parking lot is 504 said yes. Find the width of the removed strip
Answered by Penny Nom.
$9.50 in coins 2014-01-15
From dana:
you have $9.50 in coins. it has twice as many nickels as quarters and three times as many dimes as nickels. how many of each coin do you have?
Answered by Penny Nom.
Five cubes 2014-01-15
From Bob:
Rick has five cubes. When he arranges them from smallest to largest, the difference between the heights of any two neighbouring cubes is 2 cm. The largest cube is as high as a tower built from the two smallest cubes. How high is a tower built from all five cubes?
Answered by Penny Nom.
Is this operation associative? 2014-01-14
From patrick:
Associative test: Can you explain the following to me?

Is the following operation associative?: x*y=x+y+1
1) x*(y*z)=x*(y+z+1)=x+(y+z+1)+1=x+y+z+2
2) (x*y)*z=(x+y+1)*z=(x+y+1)+z+1=x+y+z+2

The answer is yes as 1) = 2)

My specific questions are:
1) How x*(y*z)=x*(y+z+1)=x+(y+z+1)+1 ?
2) How (x+y+1)*z=(x+y+1)+z+1?

Thank you!!

Answered by Penny Nom.
4x^3-7x^2-2x 2014-01-10
From Alvi:
4x^3-7x^2-2x
Answered by Penny Nom.
conical lamp stand/staved wood 2013-12-07
From Henry:
need to make lamp stand that is wooden staved; need it to be 25 inches at bottom and 10 inches at top; need to know angles for staves to be cut; the lamp stand will be rounded on a lathe and will be 40 inches tall John Lucas built one and it is pictured on his web page. thank you for any help/direction; I checked out the answered for cone shaped objects on your page but didn't find what I could use. thanks again. Henry--woodturner, parent teacher student . . . . .
Answered by Harley Weston.
f(x)= a+b*lnx 2013-12-04
From Kevin:
How do I find the curve, algebraically on paper, for f(x)= a+b*lnx with data points (6,1303) (9,1639). I can do it with LnReg on a calculator but I want to know how to get the same results or close to the same results by working it out on paper. When I run it through the LnReg on the calculator I get, f(x)=181.792+828.678 Ln x. How did it achieve this solution?
Answered by Penny Nom.
Factor 2013-12-01
From Rosa:
How do you factor 9x^2 - 12y^2?
Answered by Penny Nom.
Problem solving 2013-11-26
From Moira:
Amy has £5 more than Ben. Ben has twice as much as Chris. In total they have £20 how much do they each have
Answered by Penny Nom.
A concrete walk around a pool 2013-09-18
From poornima:
circular Swimming pool is surrounded by a concrete walk 4 feet wide, if the area of the walk is 11/25 of the area of the pool, then the radius of the pool in feet is
Answered by Penny Nom.
A word problem 2013-09-05
From Thomas:
I'm trying to help my 9th grade child with their homework. The problem is the Length of a rectangle is twice as long as it is wide. The perimeter is 68m. Find It's length and width
Answered by Penny Nom.
An algebraic equation 2013-08-28
From walter:
i'm a parent helping my daughter can you do a step by step on this equation been out of school for years thanks (4w-28)+(11w+13)=180 thanks
Answered by Penny Nom.
GST and PST 2013-07-22
From Bev:
Total sales revenues are 116391.38 this amount includes 5% GST and 80% of this amount includes a 7% provincial tax. The other 20% is PST exempt. GST is included in all. How do I figure the PST I owe?
Answered by Penny Nom.
Simultaneous equations 2013-07-10
From Warren:
solve this simultaneous equation:
xy=4
2x+3y=14

Answered by Penny Nom.
{(1+x)^1/3-1/3X(1+x)^-2/3}/(1+x)^2/3 2013-06-17
From STEPHEN:
{(1+x)^1/3-1/3X(1+x)^-2/3}/(1+x)^2/3
Answered by Penny Nom.
A silver figurine 2013-05-03
From Renee:
I have a silver figurine, the density of silver is 10.5g/cm3. if the figurine weighed 149 g. to determine the volume, he dropped it into a cylindrical glas of water. the diameter of the glass is 6 cm and the figurine was pure silver, by how much should the water level in the glass rise?
Answered by Penny Nom.
A word problem 2013-05-03
From Shaneaya:
Keikio spent the day bird watching and counted 34 more birds in the morning than in the afternoon. If she counted a total of 76 birds, how many birds did she count in the afternoon?
Answered by Penny Nom.
How do i reverse this formula? 2013-04-20
From MK:
I have 22,000,000 worth of gold and wants to want to resell the gold back with a profit. Each transaction of gold will have a transaction fee of 15%, so if i resell the gold back at 22,000,000, I will only get back 18,700,000 after 15% transaction cut. The current formula is X-15%X = Y where X = 22,000,000 and Y = 18,700,000 Will u be able to come up with a new formula if we don't know the value of X but we know the value of Y
Something like this X-15%X = 22,000,000. How do i reverse this formula to find the value of X
Thanks

Answered by Penny Nom.
An algebraic exercise 2013-03-18
From autumn:
given: f(x)=x/(x^2+2) determine: [f(x-h)-f(x)]/h
Answered by Penny Nom.
Driving a rental car 2013-03-13
From teresa:
Driving a rental car x miles cost y= 0.25x + 25 dollars.

(a) How much would it cost to rent the car but not drive it?
(b) How much does it cost to drive the car 1 additional mile?
(c) What is the y-intercept of y= 0.25x + 25? What does it represent?
(d) What is the slope of y= 0.25x + 25? What does it represent?

Answered by Penny Nom.
What is the subtotal & GST? 2013-03-04
From REBECCA:
Hi, I have a total of $300. What is the subtotal & GST? How do I work backwards?
Answered by Penny Nom.
Swimming upstream and downstream 2013-02-15
From Kimberly:
a beaver swims down to its lodge 4 times faster than it swims upstream. how many times faster is its speed in still water than the speed of the stream?
Answered by Penny Nom.
Chocolate bars and puzzles 2013-02-15
From sandy:
Alan bought a box of puzzle and two bar of chocolate for $28.
Jan bought 3 boxes of puzzle and a bar of chocolate similar to Alan for $69.
a) Find the cost of a box of puzzle
b) Find the cost of a bar of chocolate

Answered by Penny Nom.
A word problem involving toys 2013-02-14
From sandy:
Each boy gets 5 toys.Each girl gets 3 toys.There are 150 pupils.
The boys had 74 more toys than girls.
How many boys?
How many girls?

Answered by Penny Nom.
A cubical shipping container 2013-02-12
From Chantel:
The length, width and height of a cubical shipping container are all decreased by 1.0 ft, thereby decreasing the volume of the cube by 37 ft3(feet-cubed) What was the volume of the original container? (recall: volume of cube with a side length x=x3) Define the variable, write a quadratic equation and solve by factoring.
Answered by Penny Nom.
Brian is x years old 2013-02-06
From Sabrina:
Brian is x years old.
Peter is 4 years older than Brian.
Amy is 2 years younger than Brian.
The total of their ages is 26 years.
Work out the value of x.

Answered by Penny Nom.
7x-1/4-1/3[2x -1-x/2]=19/3 2013-02-03
From M:
7x-1/4-1/3[2x -1-x/2]=19/3
Answered by Penny Nom.
The area of a rectangle 2013-01-29
From Bali:
if the length of the rectangular field is decreased by 5m, its area is decreased by 70 sqm. if the width is increased by 6m, the area is increased by 180 sqm. determine the length and width of the rectangular field?
Answered by Penny Nom.
A 12 acre parcel of land 2012-12-28
From John:
What are the dimensions of a 12-acre parcel if it is 3 square acres deep and 4 square acres wide?
Answered by Penny Nom.
Increasing the size of a gift box 2012-12-27
From Tehmas:
The dimensions of a gift box are consective positive integers such that the height is the lowest integer and the length is the greatest integer. If the height is increased by 1cm, the width increased by 2 cm, and the length increased by 3 cm, then a larger box is constructed and the volume is increased by 456 cm^3. Determine the dimensions of each box.
Answered by Harley Weston.
A parabolic bridge 2012-12-09
From Elizabeth:
1) The figure below shows a bridge across a river. The arch of the bridge is a parabola and the six vertical cables that help support the road are equally spaced at 4-m intervals. Figure B shows the parabolic arch in an x-y coordinate system, with the left-end of the arch at the origin. As indicated in Figure B, the length of the leftmost cable is 3.072 m.

I'm suppose to find the (x-h)^2=-4a(y-k) equation for this word problem and I really do not know where to begin.

Afterwards, I need to find the lengths of the other cables and the maximum height of the arch of the road as well which I am very confused about

Answered by Penny Nom.
An arrow is shot straight up in the air 2012-12-06
From heather:
The skywalk is a glass bottomed platform that hangs over the edge of the Grand Canyon and is suspended 4000 feet above the floor of the canyon. If a tourist shot an arrow straight up in the air from the observation platform, the arrow's height could be modeled by H(t) =-16^2 + 240 + 4000 where H(t) is the height of the arrow above the canyon floor in feet t seconds after being shot, t>0.
(a) Find H(5) and explain its meaning

Answered by Penny Nom.
Factor (6a^2-5a+1)(8a^2-6a+1)(12a^2-7a+1) 2012-11-16
From bailey:
Factorise f(a)=(6a^2-5a+1)(8a^2-6a+1)(12a^2-7a+1) thus find [f(a)]^1/2
Answered by Penny Nom.
The sum of a number p and twenty-one 2012-10-30
From mayra:
the sum of a number p and twenty-one
Answered by Penny Nom.
Introductory algebra 2012-10-30
From kevon:
if x = 7 is used in the expression 2x + 5 what is the output
Answered by Penny Nom.
Forming an algebraic expression 2012-10-27
From ashley:
A sidewalk has a constant width W and comprises several short sections having lengths 12, 14,and 10 feet. How do I find the simplified expression that gives the number of square feet of sidewalk?
Answered by Penny Nom.
A label to cover a plastic cup 2012-10-23
From Kevin:
I'm trying to make a label to cover the entire outer area or a plastic cup. I know there must be a way to figure out the dimensions needed, but I can't seem to figure it out. The circumference of the bottom of the cup is 21.4cm and the circumference at the top of the cup is 29.8cm. The cup is 14.5cm tall. What should the height of the arc from the plane connecting the two ends of the 21.4cm arc. I attached a diagram where x is the value I'm looking for. I'm guessing there is some simple relationship between the length of a line and the arc needed to turn that line into a perfect circle, but I don't know what it is. Can you figure this out and share it with me? Thanks.

-Kevin

Answered by Penny Nom.
Solving for a/b 2012-10-21
From Jenny:
If a + 3b is equal to 175% of 6b, what is the value of a/b?
Answered by Penny Nom.
Passing out graph paper 2012-10-18
From Andrea:
mr karnowski has a stack of 125 sheets of graph paper.He is passing out g sheets to each of his students. What is the expression for the number of sheets he will pass out to the first 5 students?
Answered by Penny Nom.
A tank with an inner walled compartment 2012-10-12
From don:
I have a tank 20 feet diameter, 19' 8" tall with an inner walled compartment that has a 7' 6" radius arc with in the tank. I need to figure out the volume of the inner area and the volume of the larger area.
Answered by Harley Weston.
A problem that yields a linear equation 2012-10-07
From Eshraj:
one fourth of a whole number is three more than one fifth of the next whole number.find the whole number
Answered by Penny Nom.
Find an algebraic expression 2012-10-03
From Karen:

Question from Karen, a parent:

Find an algebraic expression relating the input and output.

input output
   
3 1
9 3
15 5
21 7
27 9

Answered by Penny Nom.
Solve 8t-r=12t for t 2012-09-29
From Monejah:
I need help solving 8t-r=12t for t
Answered by Penny Nom.
The dimensions of a rectangle 2012-09-12
From Becky:
A rectangle has an area of 48 square inches and perimeter of 32 inches. Find the dimensions of the rectangle.
Answered by Penny Nom.
(2y+1)/3=(1y-1)/2 2012-09-06
From jenna:
(2y+1)/3=(1y-1)/2
Answered by Penny Nom.
The area of a rectangle 2012-08-31
From mario:
The area of a triangle when on side is x and the other is x+6
Answered by Harley Weston.
Making a wind sock 2012-08-28
From John:
I am trying to build a wind sock and need to be able to lay the shape out on cloth. I need the wind sock front opening (diameter) to be 3 1/2" and the rear opening diameter to be 1". The windsock needs to be 9 1/2" long. I tried using the example of the person trying to make a crayfish trap but got confused and could not figure out my numbers. Any help would be greatly appreciated.

Thanks

John

Answered by Penny Nom.
A tapestry rod on a curved wall 2012-08-14
From Marlyn:
I have a curved wall with a radius of 6'. I am trying to have a 36" rod made to hang a tapestry and need to figure out the degree measure of the arc. Can you help me please?
Answered by Penny Nom.
The difference between two numbers is 7. 2012-08-09
From May:
The difference between two numbers is 7. When the two numbers are multiplied the answer is 60. How do I work out the two numbers,I tried solving for one unknown and substitution and can not do it. Pleas show me how to do this.
Answered by Penny Nom.
A 10 inch circle using 2x4s 2012-05-19
From Ralph:
I want to form a 10" circle with 4"high pieces of 2 x4's. If each 2x4 piece sit next to each other,What degree would I have to cut each side of the 2x4's, and how many would I need to form a 10 inch circle. I know there is a formula for this out there somewhere.
Answered by Harley Weston.
Measuring the liquid in a horizontal tank 2012-02-28
From Philip:
I have a steel gas tank that is 3' dia X 5' length. The total volume is 1000 litres. But how much is left when I use a stick and measure 6" from the bottom or 12" or 24" ?? Is there a formula to use for this task?

Thanks.

Answered by Harley Weston.
Figures with dots 2012-02-15
From A:
Okay so my daughter came home and had this question for homework from her teacher. I have no idea what it means HELP!!
The question shows a figure that is a square. It has 5 dots on the top, bottom and on the sides looks similar to this:
. . . . .
.       .
.       .
.       .
. . . . .
they want to know 3 more ways to show the number of dots in the figure. They have already shown one with just counting them and the other one that they showed was circling the four corner dots and then taking the dots in between with a number sentence set up like this: 4+4*(5-2) = 4+4*3= 16. They also besides giving 3 ways want us to write a number sentence and an algebraic sentence to show the number of dots in the figure with "n" sides. Please help! So confusing ! I don't know if my square turned out or not but it should be 5 across the top and bottom and on the sides and no dots in teh middle it is just the outside.

Answered by Penny Nom.
Building a tipi 2012-01-29
From Lacy:
Hi there! We are building a tipi for our children. We want to build a large one about 15ft tall with a base of about 15 feet diameter. I am trying to figure out how much canvas we need to accomplish this. I graduated about 20 years ago and am struggling. Please help if you can.
Answered by Penny Nom.
-8(3x+4)+6x=4(x-8 )+4x 2012-01-18
From bianca:
-8(3x+4)+6x=4(x-8 )+4x
Answered by Penny Nom.
What is the height of the triangle? 2012-01-12
From Hailey:
The area of a triangle is 77ft2. if the base is 11ft. how tall is the height?
Answered by Penny Nom.
Sum and difference of squares 2011-12-31
From Anne:
Se x e y são números reais distintos, então:
a) (x^2 + y^2) / (x - y) = x + y
b) (x^2 + y^2) / (x - y) = x - y
c) (x^2 - y^2) / (x - y) = x + y
d) (x^2 - y^2) / (x - y) = x - y
e) Nenhuma das alternativas anteriores é verdadeira.

Answered by Harley Weston.
i+i^2+i^3+i^4...+i^102 2011-12-13
From megan:
how do you solve i+i^2+i^3+i^4...+i^102
Answered by Robert Dawson.
factor 6x2-19x+3 2011-12-10
From pedro:
factor 6x2-19x+3
Answered by Penny Nom.
What does 2xy mean? 2011-12-07
From dana:
I don't know what is 2xy what does that mean? is it 2x+2y or 2x times y Please help! thank you for your time!! :)
Answered by Penny Nom.
The dimensions of a rectangle 2011-11-11
From Ivan:
The length of a rectangle is 31 centimeters less than five times its width. Its area is 72 sq centimeters. Find the dimensions of the rectangle.
Answered by Penny Nom.
3 fish 2011-10-27
From Ann:
You have 3 fish weighing a total of 21 lbs. The largest fish weighs twice as much as the second fish and the third fish weighs half the amount of the second fish. So how much does the third fish weigh?
Answered by Penny Nom.
Expanding the size of a table 2011-10-16
From Ericka:
You are working as a carpenter in an industrial Shop. A customer came to you and inquired about the size of the table which would be fitted in her room. She informed you that she had already a 1.5 x 1m table in her room but she wanted to maximize the space by adding the same amount to is length and width. She is planning to occupy a 3 square meter place on her room for her to work comfortably. She is requesting you a written recommendation before she asked to make a table. What amount should be added to both sides to maximize a 3 square meter area?
Answered by Penny Nom.
A rectangular wall 2011-09-12
From Jimi:
The area of a rectangular wall of a barn is 110 square feet. It's length is 12 feet longer than twice its width. Find the length and width of the wall of the barn.
Answered by Penny Nom.
A beginning algebra question 2011-09-10
From Tobby:
A=1/2 solve for h.area of a rectangle with b and h
Answered by Penny Nom.
A stained glass lamp 2011-07-25
From Guy:
Like Kay, I also work in stained glass, but in 3-dimensions. I am frequently asked to replicate lamp shades in stained glass where the diameter of the top is different (narrower) from the diameter of the bottom (which is wider). Some people want 5, 6, 7, 8, 10, 12, 16, ,,, n-sided shades. Is there a formula I can use to determine the width of the sides using the angle, if I remember correctly, I think it's called theta. In other words, is there a formula where I can plug in the angle which describes the arc of the circle. For instance, if someone asks for a 7 sided shade, plugging in 51.43 (360/7). I could then use that to determine the width at the top and bottom rings to create the appropriate trapezoids. I've visited a few sites so the formula looks like its a function of sin & cos but they are presented like proofs for teaching. Your site appears to want to actually answer questions without making the inquirer feel stupid.
Answered by Harley Weston.
The height of a fluid in a horizontal tank 2011-07-24
From jason:
Same set up as many others, cylindrical tank on its side, but I am interested in defining the change in volume and/or fluid level as a function of time at a constant volumetric outflow. I plan on hooking a pump to the tank so "gpms' will be constant. I have a couple different sized tanks and pumps so I want a general equation. Thanks for your help.
Answered by Harley Weston.
The length of a belt around three pulleys 2011-05-18
From Grant:
I need to calculate the belt length around these pulleys, please can you help or refer me?

Known variables
D - Large Pulley Diameter
d - Small Pulley Diameter
c - Center Distance between D and d
T - Tension Pulley Diameter
x - Horizontal Distance between T and d' Centers
y - Vertical Distance between T and d's Centers
I need to calculate the belt length around these pulleys.


Kind Regards,
Grant

Answered by Harley Weston.
What was his salary before the raise? 2011-05-16
From Annette:
Need to show all my work and round to the nearest dollar. If James received a 9% raise and is now making $102.00 per year, what was his salary before the raise?
Answered by Penny Nom.
The dimensions of a rectangle 2011-05-08
From yolanda:
If the length of a rectangle is 3ft less than twice its width and the area of the rectangle is 54ft^2,what are the dimensions of the rectangle?
Answered by Penny Nom.
(4x-5)(3x+2)=0 2011-05-04
From Corina:
solve: (4x-5)(3x+2)=0
Answered by Penny Nom.
How many students are 25 and under? 2011-05-01
From ella:
use the five steps of problem solving to answer the following question Enrollment at a local university is 8,500. There are one third as many students over the age of25. there are 25 and under. how many students are 25 and under and how many are 25 and older(p 5)
Answered by Penny Nom.
The area of a rectangle 2011-04-30
From Hayley:
A rectangles width is exactly twice its length. The area of the rectangle is 450 square feet. What are the length and width of the rectangle?
Answered by Penny Nom.
Deciding where to bowl 2011-04-29
From ella:
the bowling alley on wedge street charges $5.50 for shoe rentals plus $2.50 per game. lawerence bowing alley charges $4.00 shoe rentals and $3.00 per game . how many games should they bowl to make wedge street cheaper than lawerence street
Answered by Penny Nom.
x/6 + 4/5 = x/9 2011-04-10
From Khadija:
x/6 + 4/5 = x/9
Answered by Penny Nom.
The perimeter of a rectangle 2011-04-04
From aspen:
the perimeter of a rectangle is twice the sum of its length and its width. the perimeter is 40 meters and its length is 2 meters more then twice its width. what is the length?
Answered by Penny Nom.
[(90+36-4) ÷ 2] x 15 = 2011-03-30
From ken:
[(90+36-4) ÷ 2] x 15 =
Answered by Penny Nom.
6(M-1/9)=55/12 2011-03-15
From Colleen:
6(M-1/9)=55/12
Answered by Penny Nom.
Three consecutive odd numbers 2011-03-11
From George:
Write an equation three consecutive odd numbers whose sum is 75
Answered by Penny Nom.
Calibrating a conical tank 2011-02-05
From Bill:
Hi, I have a round tank with tapered sides where I know the diameter at the top and bottom. Is there a formula I can use to calculate the volume by measuring from the bottom up the side (at the angle of the side) to any given point? Thanks, Bill
Answered by Stephen La Rocque and Penny Nom.
A fence around a water tank 2011-02-01
From Heath:
I am building a fence around a water tank. the fence is to be in the shape of a normal octagon. The tank has a circumference of 57 ' 6''. I would like the fence to be 3 ft from the tank at the skinny point . How would I calculate(for the simple guy) where to set each of my 4x4 posts at the 8 corners. Any help would be greatly appreciated.
Answered by Harley Weston.
The area of a rectangle 2011-01-31
From T.S.:
for which value of w does the rectangle below have an area of 864 square units if sides are width=w+15 and length=w-15
Answered by Penny Nom.
A rectangle 2011-01-16
From meg:
if a rectangle has a length that is 3cm less than 4 times its width and its perimeter is 19cm, what are the dimension?
Answered by Penny Nom.
2x = 2+2x 2011-01-15
From james:
2x=2+2x can you help me find what x epauls?
Answered by Penny Nom.
Matt is thinking of two numbers 2011-01-14
From mary:
Matt is thinking of two numbers. One number is 9 more than the other number. The sum of the numbers is 45. What are the numbers?
Answered by Penny Nom.
An algebra problem with fractions 2010-10-16
From Morgan:
Hi my math teacher has given us this question for our grade nine math class and I always get confused about multiple x algebra, could you help me?

6x+4        3x+2
------   =  ------
  3               5



I have no problems until it comes to combining integers, or x's

Answered by Penny Nom.
A linear system 2010-10-15
From Sandy:
HARD!! algebra (Linear Systems) question? An Airplane cruises at 120km/h in still air. One day, when the wind was blowing steady from the west, the airplane travelled west (flying into the wind) and landed at its desination after 1.5h. The airplane's return trip (flying with the wind) was only 1.0 h long. On both legs, the airplane travelled at its cruising speed. (meausred with respect to the air surrounding the airplane).

Part A
Write a System of equations that would allow you to determine the speed of the wind blowing that day and the distance travelled each way

Part B
When solving the system of equations graphically, what equations would you enter into the equation editior (to make graphs on your calculator)

Part C
Indicate what the X and the Y represent

Answered by Stephen La Rocque.
2x − 1 ( 1 − 2x) = 1 ( 2x − 2x) 2010-10-08
From MC:
Solve 2x − 1 ( 1 − 2x) = 1 ( 2x − 2x) for x

I am befuddled! Thank you for any explanation you can give that show me the steps. I know the order in which I should tackle it, but am so confused regarding how to assume x. Thank you!

Answered by Penny Nom.
20% profit on the selling price 2010-09-24
From ata:
My cost is 100 and I want to make 20% profit on my selling price (not 20% of the cost). So, what should be calculation formula? Let's say my cost is 493.71 and I want to make 53.28% profit on the selling price; so how to determine the selling price?
Answered by Penny Nom.
A reverse PST calculation 2010-09-23
From Manning:
Hi, I'm a manufacturing contractor in Saskatoon. The way to calculate PST for us is using total contract amount x 70% then 5%. For example, contract amount of $100 x 70% x 5% = $3.5 (PST). The subtotal is using contract amount + PST. $100 + $3.50 = $103.50.

If you only know the subtotal is $103.50 including PST, how do you reverse calculation to get the contract amount?

Thank you very much for your help.

Answered by Harley Weston.
An algebraic equation with fractions 2010-09-10
From leah:
If 6/x - (x-1)/2 = 4 then the LCD is 2x, right? so you get 12/2x - (x2-x)/2x = 8x/2x, right? then what do you do with the 2x when you turn it into a quadratic equation?
Answered by Penny Nom.
Algebraic Equations 2010-08-10
From Justin:
the amount of money that Mika has if she has some quarters

how fast Rya runs if she runs 5 mi/h slower than Danae
please help,
thanks, justin

Answered by Robert Dawson.
A shrunken car 2010-07-31
From Diana:
Car manufacturers have outdone themselves. They have discovered how to shrink a car to make parking easier. At the touch of a button, a car shrinks by 2 feet in the width and 4 feet in the length. The original car base is 24 square feet longer than that of the shrunken car. If the original cars length is twice the width, find the dimensions of both cars. Identify the unknowns carefully.
Answered by Penny Nom.
An octagon shaped bench 2010-07-09
From rob:
i am trying to build a octagon shaped bench to fit inside a 69 inch round hot tub so that the tip of each point touches the edge of the circle where it will be fastened.
Answered by Stephen La Rocque.
An algebraic fraction 2010-06-28
From luis:
3x-2//4-3x=12
Answered by Stephen La Rocque.
Which values of x satisfy (x-2) / (x+4) <7 2010-05-23
From Rocco:
Which values of x satisfy (x-2) / (x+4) <7
Answered by Harley Weston.
(x+1)(x+2)(x+3)/(x+1)(x+2) 2010-05-19
From Nazrul:
Simplify : (x+1)(x+2)(x+3)/(x+1)(x+2)
Which answer is correct:
(i) x+3
(ii) (x+2)^2(x+3)
Please help me.

Answered by Harley Weston.
A loop 2010-05-05
From Amir:
what is loop theory in algebra?
Answered by Chris Fisher.
How many terms are in the simplified expression? 2010-04-29
From Anu:
The expression (x+y+z)^2006 + (x-y-z)^2006 is simplified by expanding it and combining like terms.
How many terms are in the simplified expression?

I've spent hours on this. pls help!!!
Also, is answering these questions a paid job for you or is it volunteer work?
I'd like to join an ask and answer site for math, (when I grow up)
How does one go about doing that?

Answered by Robert Dawson.
The width of a picture frame 2010-04-28
From Daniel:
The frame of a picture is 28 cm by 32 cm outside and is of uniform width. What is the width of the frame if 200 cm ^2 shows?
Answered by Penny Nom.
A rectangular garden 2010-04-25
From Billy:
Tanisha wants to make a rectangular garden with a perimeter of 38 feet. What is the greatest area possible that tanisha can make the garden?
Answered by Penny Nom.
Algebraic fractions 2010-04-22
From rory:
3x/(x²-64)+4/(x²-6x-16)=
Answered by Robert Dawson and Harley Weston.
The length and width of a table 2010-04-19
From Linda:
The length of the top of a table is 2 feet longer than 4 times the width. The area is 72m squared. Find the dimensions of the table.
Answered by Penny Nom.
Four less than three times a number is 14 2010-04-16
From David:
My daughter has two questions that we could use some help with.

First ?
Four less than three times a number is 14

Secound?
The quotient of -15 and w

Answered by Penny Nom.
A rectangle problem 2010-04-12
From Charlene:
The ratio of the width to the length of a certain rectangle is equal to the ratio of its length to the difference between its length and twice its width. If the area of the rectangle is 25cm^2, then what are its dimensions? Present a non- algebraic solution?
Answered by Tyler Wood.
Coins in 3 parking meters 2010-03-27
From Zena:
There are 227 coins altogether in 3 parking meters. The second meter has 37 more coins than the first meter. The third meter has twice as many coins as the second. How many coins in each meter?
Answered by Penny Nom.
y = kx 2010-03-26
From harold:
if y =kx and y =4/15 and x=2/3, what is value of k
Answered by Penny Nom.
Selling lettuce 2010-03-11
From Mike:
Day 1 started with 80 heads of lettuce and x were sold.
Day 2 started with two times the number of heads of lettuce that was left over. Again, only x were sold.
Day 3 started with 3 times the number of lettuce from what was left over from day 2 and all was sold.
What was the # of lettuce sold on each day?

Answered by Penny Nom.
Log[f(x)] 2010-03-08
From sourabh:
What is the number of solutions of the equation 9 x 2 - 18 x + 5 = 0 for x, such that the expression log10[(x+1)(x+2)] exists?
Answered by Harley Weston.
Redwood Park and Glendale Park 2010-03-05
From Shaun:
Question from Shaun, a student:

Redwood and Glendale Parks are each surrounded by 100 yards of fencing. The are of Redwood Park is 25 square yards more than the area of Glendale Park. If both parks are rectangles, what is the measurement of each rectangle?

Answered by Harley Weston.
Standard form 2010-03-05
From Laura:
How do I change 3y=4x+1 into standard Ax+By=C standard form. This is my son's schoolwork. I homeschool him. I got the answer to the problem as 3y=4x+1, but it needs to be written in standard form and I know it needs to read 4x-3y=-1, but how do I get that answer into standard form. I need to know how to explain it to him. Can you show me? Thank you so much.
Answered by Penny Nom.
The dimensions of a rectangle 2010-03-03
From Kirsten:
A rectangle is twice as long as it is wide. Its area is 128 square meters. What are the dimensions of the rectangle?
Answered by Tyler Wood.
An algebraic equation with fractions 2010-02-19
From Ingrid:
What is the solution set of the equation x over x plus 4 = 1 over x plus 3 = 28 over x to the 2nd power - x- 12?
Answered by Penny Nom.
The center of a rectangular room 2010-02-16
From Diana:
Consider a rectangular room, 15 feet wide, 30 feet long and 12 feet high. What is the exact distance from any of the 8 corners of the room and its geometric center? Can you write a generic formula for such a distance? And if we keep the same proportions on all dimensions, can you write an expression for the same distance as a function of the floor perimeter?
Answered by Penny Nom.
The dimensions of a flag 2010-02-15
From dawn:
a state flag was twice as long as it was wide and had an area of 288 sq inches, what were its, dimensions
Thanks

Answered by Penny Nom.
Mixing oil 2010-02-03
From Andrew:
A dealer wishes to produce 300 gallons of oil worth 40cents a quart. He has oil worth 36cents a quart and oil worth 52cents a quart. How many quarts of each should he mix?
Answered by Penny Nom.
divide the product of 'x' and '3y' by '2z' 2010-01-19
From shradheya:
write all fall using variable ,number, and basic operation

1. divide the product of 'x' and '3y' by '2z'.

Answered by Penny Nom.
Setting up an algebraic equation 2010-01-18
From jerry:
the 1st of 4 books has twice as many pages as the 3rd. The 2nd book has 30 pages less than the 1st. The 4th has 42 pages more than the 1st and 2nd books combined. there are 567 pages in total. How many pages are in each book.
Answered by Penny Nom.
4x +3/4 = 1/4 2009-12-12
From Lynda:
4x +3/4 = 1/4

I know the answer is -1/8 but I don't know how to show the process of getting the answer

Answered by Penny Nom.
Books on a shelf 2009-10-25
From Julie:
a bookshelf is 36 inches long and contains (x) books each (n) inches thick. If each book half an inch thinner, the shelf would hold six more of the same book. What is x/n
Answered by Penny Nom.
f(x + h)-f(x) 2009-09-07
From Marie:
For each function defined as follows, find a.) f (x+h) b. f(x+h)-f(x) c. [f(x+h)- f(x)]/h.

Problem: f(x)= -1/ x^2.

Answered by Harley Weston.
The area of a rectangle 2009-08-21
From roxanne:
the dimensions of the rectangle: 3x length and 4x width. What is the area of the rectangle in square units?
Answered by Penny Nom.
Pre-Algebra Resources 2009-07-16
From M:
What is the best pre algebra book that is inexpensive and will really help me to understand pre algebra?
Answered by Janice Cotcher.
Finding the Line Joining Perpendicular Feet 2009-07-16
From mukulu:
Please help I've been tryng to search in your data but i failed to get the solution Find the equation of a straight line joining the feet of the perpendiculars drawn from the point A(1,1) to the line 3x-3y-4= 0 and 3x+y-6=0.
Answered by Janice Cotcher.
The product of two consecutive integers 2009-07-13
From kelly:
the product of two consecutive integers is 9 less than the square of the second integer
Answered by Penny Nom.
x^3+1/x^3=18*sqrt(3) 2009-07-08
From Nazrul:
If x^3+1/x^3=18*sqrt(3) then how can I evaluate the value of x+1/x? Thank you for your help.
Answered by Robert Dawson.
Solving an Algebraeic Equation with Fractions 2009-06-04
From olivia:
solve for x (3x-1)/4 + (x+3)/6 = 3
Answered by Janice Cotcher.
Solve 18(C-3)=162 for C 2009-05-29
From Alicia:
18(C-3)=162

adult student returning to school after 20 years and then never had it in school. Just doesn't click yet

Answered by Penny Nom.
Winding paper after a break 2009-04-10
From Olen:
Question from Olen:

I work in a paper mill and have been handed the task to search for a formula to determine how much paper needs to be added to a parent roll to make up the difference at the winder. (Ex. The spool diameter at the reel is 18.25" we measure roughly 33.5" to make two 58" rolls in the winder. If the is a paper break and the roll diameter in the winder is 30" how much do I add to a single parent roll (22" roughly) to make one 58 " and the 28" needed at the winder. I would appreciate any help to complete this task. I would like to be able to build a chart that operators can refer to based on what is needed. Thank you.

Answered by Harley Weston.
A five game average of 148 2009-04-01
From Shelly:
(128 + 145 + 139 + 157 + x) divided by 5 = 148 matches the situation stated below:

A person bowled four games and scored 128, 145, 139, and 157. Find x, the score the person would need to have in the fifth game to average 148 for all five games.

How do I solve this and how am I supposed to add x like it says in the equation?

Answered by Penny Nom.
x^2/5 - 4/5 = -3/5x 2009-03-25
From Lorrie:
x^2/5 - 4/5 = -3/5x
Answered by Penny Nom.
Fertilizer in a bin 2009-02-03
From Todd:
Hello I am looking for a formula to figure out the fertilizer volume in a hopper bottom bin not only when it is full but part full as well. When you are filling it is heaped up in the middle to make a cone and when you are emptying the bin the cone is inverted so it would be nice to be able to quickly figure out the tonnes partly filled and when full.
Lets say the bin is 32 feet high from top of bin where you fill to the bottom where the product goes out and it is 16 feet in diameter. I know how to calculate the cylinder it is the cones on the top and bottom of the bin I have the main question on.

Answered by Harley Weston.
Calculating markup 2009-02-03
From Tonya:
I'm trying to figure out the formula for the following, I have a product that I want to receive a certain profit, but with this product I also have to pay a percentage of commission to someone. I need to know how to calculate my new selling price taking into account the markup percentage, but I don't want my cut to be lowered. I have calculated the markup percentage to get my new selling price with the commission, but it lowers my cut once I pay the commission and I don't want it to do that. Thanks
Answered by Penny Nom.
Find area of the rectangle in terms of L. 2009-01-27
From Arlen:
The length of a rectangle is L. The width is 8 less than the length. Find area of the rectangle in terms of L.
Answered by Penny Nom.
Solve (3x-2)/4 - 3x = 12 2009-01-26
From michelle:
3x-2/4 - 3x = 12
Answered by Penny Nom.
The whole number that precedes a number w+3 2009-01-17
From Morgan:
write a variable expression for "The whole number that precedes a number w+3"
Answered by Penny Nom.
An expression for q quarters and n nickles 2009-01-16
From Morgan:
a variable expression for:

"The value in cents of q quarters and n nickels"

Answered by Penny Nom.
For which integers a, b, c does a - b - c = a - (b - c)? 2009-01-13
From Johanna:
For which integers a, b, c does a - b - c = a - (b - c)?
Answered by Penny Nom.
Eric bought 3 sweaters and 1 pair of pants 2009-01-01
From maya:
eric bought 3 sweaters and 1 pair of pants. Each sweater cost d dollars and the pants cost 20 dollars. Eric spent a total of 95 dollars. The equation below can be used to determine the cost of each sweater. what is the answer?
Answered by Penny Nom.
Solve for x 2008-12-16
From Melissa:
I have a test tomorrow and I'm hoping you can help me before then. I can never seem to solve the "RESOLVE X" problems, or in french resous pour x/
They look like this. 3x+2\6=2x-5\3. I only understand NOTHING from that. Another equation is 2(x+1)=3(x+2).

Answered by Robert Dawson and Penny Nom.
Determining the Unknown Numbers 2008-12-06
From jacob:
the sum of two numbers is 45 and their difference is 15. what are the two numbers?
Answered by Janice Cotcher.
An algebraic exercise with fractions and radicals 2008-11-18
From Rubén:
Question from Rubén, a teacher:

I have the ecuation:

U = [yb / (ab+a^2)]^1/2 + [ya / (ab+b^2)]^1/2

I know for sure this reduces to

U = [y (1/a + 1/b)]^1/2

but I cannot find a way to get into that result!

Thanks

Answered by Rubén Osuna.
Equations with fractional powers and roots 2008-11-12
From David:
Find all real solution to each equation.

x^-2/3 = 9 how do you do this if its to the power

x + 1= Square root (3x +13) can you explain how to get an solution with square root?

Answered by Penny Nom.
Algebraic equations with fractions 2008-11-07
From John:
Solve each equation. Check each solution.
1/8+5x/x+2=5/2
10/2y+8 - 7y+8/y(squared)-16 = -8/2y-8

Answered by Harley Weston.
Solve the equation 2x + y = 6 for y. 2008-11-07
From fatiuma:
Solve the equation 2x + y = 6 for y.
Answered by Penny Nom.
Algebraic fractions 2008-10-06
From Kayla:
(x^2-9)/x times (x^3-4x)/(x^2+5x+6)
Answered by Harley Weston.
Adding fractions 2008-09-17
From amanpreet:
please can you help me to simplify:

2(c-3) / 7 + 3(4c-2)/2

Answered by Penny Nom.
Two Step Equation 2008-09-17
From calea:
i really dont now what two-step equation means?
Answered by Janice Cotcher.
Cows, chickens, heads and feet 2008-09-01
From marina:
Iam helping my 6th grader son solve this problem. I found similar problem in the Q&A.I understood the simple answer without using an equation. I want to solve this using the equation.This is the question about chicken and cows. There are cows and chickens in the farm. There are 65 heads and 226 legs. How many chicken are there?
Answered by Harley Weston.
v-4(4-v) = -2(2v-1) 2008-08-29
From c:
Please explain all the steps to solving this

v-4(4-v) = -2(2v-1)

Answered by Penny Nom.
An algebraic expression 2008-08-29
From Antonio:
write a variable expression for 9 less than k
Answered by Penny Nom.
Algebra 2008-08-10
From Reasat:
How to understand algebra?
Answered by Stephen La Rocque.
1/X-2 + 3/X+3 = 4/X^2+X-6 2008-08-06
From Molly:
Hi, I'm beginning pre-calc this semester, and for practice over the summer, the teacher has given us a review packet with algebra 2 stuff. Well, I've forgotten most of the fraction stuff on it. Would you please help? Here's the question:

1/X-2 + 3/X+3 = 4/X^2+X-6

Answered by Penny Nom.
Solving with the Natural Log 2008-07-28
From CB:
I am trying to solve lnx +3ln2 = ln 2/x. Every way I've tried when I go to plug in the solution it doesn't come out right.
Answered by Penny Nom.
Solving for Two Variables 2008-07-24
From Mary:
can you help, 2/35= 1/x +1/y ; x
Answered by Janice Cotcher, Victoria West and Harley Weston.
Four Positive Integers 2008-07-20
From william:
let a, b, c and n be positive integers. If a+b+c=(19)(97) and a+n=b-n=c/n, compute the value of a.
Answered by Janice Cotcher.
multiply = 3a - 3b by a+b 2008-07-15
From chaleen:
multiply = 3a - 3b by a+b
Answered by Penny Nom.
x=(-14x+16)/x-8 2008-06-11
From Kory:
Solve the equation.
x=(-14x+16)/x-8

Answered by Penny Nom.
Substitution into a formula 2008-06-02
From beth:
Given q(x) = 7xsquared + 9, find q(-7)

These were the choices I had for the answers to this problem
1.) 343
2.) -40
3.) 334
4.) -89
5.) 352

Answered by Penny Nom.
1+1/1+1/x 2008-05-21
From nadean:
1+1/1+1/x
Answered by Penny Nom.
The sides of a rectangle 2008-05-16
From bill:
The area of the rectangle is 170 sq.ft. the width is unknown and the length is the width plus 7 ft. I set it up like this w(w+7)=170 and this is where I get lost. Thanks, Bill
Answered by Penny Nom.
The rectangle with smallest perimeter 2008-04-01
From nicole:
when given an area,squares are the rectangles with the smallest perimeter write the length and width of the rectangle with the smallest perimeter
Answered by Penny Nom.
Algebraic fractions 2008-02-21
From sergio:
how to simplify
(x^2 +5x+6)/(x^2 - 4) x (x^2 -5x+6)/(x^2-9)

Answered by Stephen La Rocque.
18x=6(x*x) 2008-02-20
From peyton:
18x=6(x*x)
Answered by Stephen La Rocque.
Subtracting two algebraic fractions 2008-02-17
From Willie:
Subtract and express the answer in simplest form. 5x-3/6-x+3/6
Answered by Penny Nom.
The angles in a parallelogram 2008-02-11
From lauren:
if the sum of the measures are 360 in a parallelogram. a and d are same measures, c and b are same too. If c is twice the measure of a what are all the angles?
Answered by Penny Nom.
A 20 foot board is cut into two pieces 2008-02-11
From Sherry:
A 20 foot board is cut into two pieces. If we let x= the length of the longer piece, what is the length of the shorter piece?
Answered by Stephen La Rocque.
Sum and difference of cubes 2008-01-30
From Amanda:
It has been a really long time since I was in Algebra and I can't remember how to factor cubes such as x^3 +81 or subtracting/adding fractions with variables such as [1/(x+h)+2]-[1/x+2]. Please help!!!
Answered by Penny Nom.
8/9 + x = 9/5 2008-01-17
From nas:
8/9+x=9/5
Answered by Penny Nom.
2 divided by x plus 1 is equal to 3 2008-01-14
From chilly:
with x being the subject of the formula, what is the answer of 2 divided by x plus 1 is equal to 3?
Answered by Penny Nom.
2(x - 5) + 45 = 4 - (3 - 2x) 2008-01-09
From Kieran:
Solve
2(x-5)+45=4-(3-2x)

Answered by Penny Nom.
An algebra exercise 2007-10-22
From Math:
The number of moles of an ideal gas can be found from the formula:

PV/RT=N

solve the formula for "R", the ideal gas constant.

Answered by Penny Nom.
sin 43= y/20 2007-10-21
From stefany:
if sin 43= y/20, what is the value of y to the nearest tenth?
Answered by Penny Nom.
3x2/3 = 12/3 2007-09-28
From Shafan:
3x^2=12
Answered by Penny Nom.
The country with the most universities 2007-09-28
From j:
the country with the most universities is india followed by the united states. If india has 2649 more universities than united state and there combine total is 14,165, find the number of universities in india and the number in the united states.
Answered by Stephen La Rocque.
1/2+X=5/8 2007-09-24
From Tasha:
What is the sum of X in the problem below and explain your answer please?

1/2+X=5/8

Answered by Chris Langdon.
2x+1+x-4+4x+1 2007-09-06
From laurie:
2x+1+x-4+4x+1
Answered by Penny Nom.
13 = x/20 2007-09-06
From Melissa:
can u please help me with this question?
13 = x/20

Answered by Penny Nom.
The quotient of thirty and ten times a number. 2007-09-02
From Jenna:
I need help on How to translate a phrase into an algebraic expression: The quotient of thirty and ten times a number.
Answered by Penny Nom.
Adding algebraic fractions 2007-08-14
From John:
Ive completely forgot anything to do with the subject mentioned, so my question is straight to the point..

I need to know how to do the following problem (Preferably do not give me an answer though) (k/3k-8) - (4/k+2)

Answered by Penny Nom.
10r^2 - 35r = 0 2007-08-13
From Aranxa:
how do u solve this equation by factoring: 10r^2 - 35r = 0
Answered by Penny Nom.
If x = 2 then 4 + 5x / 4 + x = ? 2007-07-18
From Daniel:
If x = 2 then 4 + 5x / 4 + x = ?
Answered by Stephen La Rocque.
How many women and men math students are there? 2007-07-09
From Bing:
there are 38 math students. How many men and women strudents are there if the women exceeded one half of the men by 17?
Answered by Stephen La Rocque.
Algebraic EquationsAx+By=C 2007-07-02
From Juan:
The general "Standard" form of an equation is Ax+By=C. Re-Write this equation in slope-intercept (y=mx+b) form.
Answered by Harley Weston.
Algebra : 9x + 7 = 5x - 3 2007-06-15
From Allen:
I am having a problem solving for x. The problem is: 9x + 7 = 5x - 3
Answered by Stephen La Rocque.
Algebra - find the number 2007-06-04
From Fabiola:
When 4 times a number is increased by 40, the answer is the same as when100 is decreased by a number. Find the number
Answered by Stephen La Rocque.
2x + 3x = 30 2007-06-01
From Jessica:
2x+3x=30
Answered by Stephen La Rocque.
Factor Completely 2007-05-29
From Ema:
Question 1:
Factor completely. 3x(x – 4) + 5(x – 4)
Question 2:
Factor completely. x^2 + 2x + 3

Answered by Penny Nom.
y=mx+b 2007-05-29
From molly:
Im having a problem with the y=mx+b for our homework we have a table that goes from 0 to 6 , 1 to 7 and 2 to 8 and we have to make a equation of that. Please help.
Answered by Penny Nom.
x/2+3=2 2007-05-28
From Ethel:
what is the formula and the answer to x/2+3=2?
Answered by Penny Nom.
Algebra (Solve for B) 2007-05-23
From Mark:
A = .250 * (B^2*SIN(C)) / (D*COS^2(E)). I need to solve for B.
Answered by Stephen La Rocque.
(3x-2(x-3)+2)(3x-2(x-3)+2) 2007-05-17
From Jeff:
(3x-2(x-3)+2)(3x-2(x-3)+2)
Answered by Penny Nom.
Algebra problem 2007-05-10
From Sam:
What is the proper answer for this equation?
3x - 2 ( 5y (4x +2))

Answered by Stephen La Rocque.
Pre-algebra volume of a prism 2007-05-10
From Brenda:
Find the volume of a triangular prism with a base of 5 cm by 5 cm and a height of 12.5 cm. Round to the nearest tenth.
Answered by Stephen La Rocque.
Interest Rates 2007-05-09
From Julia:
Using the formula I=PRT to figure out the missing variable.
P=$9530, T=7 months, A= $9919.14

Answered by Stephen La Rocque.
What number am I? 2007-05-09
From austin:
i am a positive integer if im squared -5 is added to me. -10 is subtracted from me, and the remainder is divided by 2 the result is 7. what integer am i
Answered by Stephen La Rocque.
Who was the mathematician that united algebra and geometry? 2007-05-08
From Victor:
who was the mathematician that united algebra and geometry
Answered by Penny Nom.
Algebra 2007-04-16
From Eugene:
X over 2, -3 =0 ?
Answered by Haley Ess.
Algebra 2007-04-03
From Emily:
9,365 and 2,242 added to the difference between 255 and half of a number is 11,784. What is the number?
Answered by Haley Ess.
An equation with fractions 2007-04-03
From deanna:
x      x
-- + -- =8
9       3
need help with showing work and answer

Answered by Penny Nom.
-4x=1/5 2007-04-03
From deanna:
-4x=1/5 need help with showing work and answer
Answered by Stephen La Rocque.
Find the expression that represents it perimeter. 2007-04-01
From Amy:
A rectangle has sides of 3x-4 and 7x+10. Find the expression that represents it perimeter.
Answered by Steve La Rocque and Haley Ess.
Solve for x and y 2007-03-27
From anthony:
solve y=4x and x+y=5
Answered by Sara Ulmer.
Examples of algebra used in life 2007-03-21
From Arianna:
i need to find about 5 different examples of algebra used in life, and im not sure where to start. does anyone have any suggestions as to topics that i should choose for my project? thank you so much for all of your input. Arianna Leigh
Answered by Penny Nom.
18x - 5 = 3(6x - 2) 2007-03-19
From Lordyne:
How do I work this math problem? 18x - 5 = 3(6x - 2)
Answered by Pam Fowler and Penny Nom.
Algebra 2007-02-13
From B.Wilson:
The teacher has asked to solve the following, but I have never seen this before: 6x/2 = 4x/5
Answered by Haley Ess.
Dividing money unequally 2007-02-06
From Linda:
I have a total of $13,826.48. It is to be divided by 15 kids - 2 of the kids only get half of what the other 13 do - how much do the 13 get and how much does the 2 get? Thanks
Answered by Haley Ess and Steve La Rocque.
Nickels, dimes and quarters 2007-02-05
From Avinash:
Mary has 48 coins made up of nickels, dimes and quarters with a total value of $5.10. She has 4 more dimes than nickels and quarters combined. How many coins of each kind does she have? Use matrix to solve the system
Answered by Stephen La Rocque.
An algebra exercise 2007-01-24
From Clara:
Given x + 2y = -2 and x - 2y = 18, find:
a) x^2 - 4y^2
b) x^2 + 4y^2

Answered by Stephen La Rocque and Penny Nom.
p=2l+2w find l 2007-01-23
From Jadesola:
p=2l+2w find l
Answered by Stephen La Rocque and Penny Nom.
How do you write the equation y= 4x-2 in ax+by=c form? 2007-01-17
From Evan:
How do you write the equation y= 4x-2 in ax+by=c form?
Answered by Penny Nom.
y = 5 x - 3 2007-01-10
From Richard:
Question is the function y = 5 x - 3 for the given value of x 7 , 1 or 4
Answered by Stephen La Rocque and Penny Nom.
Products with symbols 2007-01-09
From Jennifer:
Find the product of each
(8x)(-4)
(3x)(5y)(7z)
(8x^3y)-5X^2)
(6a)(3a)(-b^2)

Answered by Penny Nom.
Tiles arranged in rows 2007-01-02
From Ann:
Some tiles are arranged in rows so that the number of tiles in each row is 8 more than the number of rows. The same number of tiles can be arranged in 3 more rows than the first pattern with 16 tiles in each row. Find the total number of tiles.
Answered by Stephen La Rocque.
6/x-2y - 15/x+y = 0.5 2006-12-21
From Emeka:
6/x-2y - 15/x+y = 0.5
Answered by Stephen La Rocque.
How many M&Ms are in a container? 2006-11-01
From Jill:
How do we algebraically figure out how many M&Ms are in a container?
Answered by Stephen La Rocque and Penny Nom.
How do you write a variable expression 2006-10-11
From Josh:
How do you write a variable expression for the following:

The amount of money Waldo has if he has $10 more than Joe and

How much weight Kirk can lift if he lifts 30 lbs more than his brother.

Answered by Stephen La Rocque.
30+7(x-1)=3(2x+7) 2006-09-29
From Art:
30+7(x-1)=3(2x+7)

I know with the distributive law I multiply 7 by x and 7 by -1 and the same thing to the right, only 3 by 2 and 7 than I am lost?
Answered by Penny Nom.

How many items must the company produce to begin to make a profit? 2006-09-24
From Devon:
The profit a manufacturing company makes can be found using the formula P=120n - n^2 - 2200 How many items must the company produce to begin to make a profit?
Answered by Stephen La Rocque.
What was the total cost of the dinners he served 2006-09-10
From Connie:
Walter is a waiter at the town diner. He earns a daily wage of $100 plus tips that equal to .15 of the total cost of the dinners he serves. What was the total cost of the dinners he served if he earned $170 on Tuesday?
Answered by Stephen La Rocque.
How much did I spend on the first trip? 2006-09-10
From Alexander:
I have $600 to spend.
I take 10 shopping trips.
Each trip I spend $10 more than the last trip.

How much did I spend on the first trip?
Answered by Stephen La Rocque.

How many marbles are there? 2006-08-30
From Cathey:
Four children are playing with marbles. At the end of the day, one child has four less than half the marbles. The second child has six more than one-fifth the marbles. The third child has one third of what the first child has and the fourth child has one less than the third child. How many marbles are there?
Answered by Stephen La Rocque.
How many total worker are in the factory? 2006-08-26
From Sherine:
One quarter of the workers at the factory are clerical, one fifth are technical, half are are administrative,and the remaining 25 are managerial. How many total worker are in the factory?
Answered by Penny Nom.
A board is cut into two pieces... 2006-07-11
From DeAnna:
A 102" board is cut into 2 pieces. One piece is two times the length of the other. Find the lengths of the two pieces.
Answered by Stephen La Rocque.
Two algebra questions 2006-07-02
From Majeedah:

I'm upgrading thru correspondence and haven't been in school for a long while, so I have no class or teacher to explain the basics to me.

I have a couple of questions:

The problem is this:

  1. f(x) = x2 - 2, Find the expression.

    Q f(-x)
    A x2 - 2

    Why is the answer not -x2 - 2?

  2. A relation f, is given by f(x) = x - 2/x. Find the expression.

    Q 2/f(3)
    A 6

    How do you get the answer.


Answered by Stephen La Rocque.
simplify 7x + 4y + 2x - 6y 2006-06-21
From Alan:
simplify 7x + 4y + 2x - 6y
Answered by Paul Betts.
The hardest algebra question you know 2006-06-08
From Steven:
could you send me the hardest algebra question you know and the work with it , i want to test a tutor. he a friend.
Answered by Stephen La Rocque.
Solving an equation without using algebra 2006-05-01
From Kenneth:
How can the following be solved without using algebra? 10 times some number is 3 times the same number plus 14.
Answered by Paul Betts.
Finding the supplementary angles 2006-04-22
From Kendra:
Angles ABC and DBA are supplementary. If m
Answered by Stephen La Rocque.
Finding c in 4x=10-cy 2006-04-19
From Jennifer:
A solution of the equation 4x=10-cy is (5,2). What is the value of c?
Answered by Stephen La Rocque.
x-7=20 and 2x+5=17 2006-04-04
From Janette:
I'm trying to help out my daughter, it has been much too long since I've been in school. The question is x-7=20 and the other question is 2x+5=17. What are the formulas?
Answered by Penny Nom.
what is 9x3h for h = 2 2006-03-25
From Dillon:
what is 9x3h for h = 2 just don't get it.
Answered by Penny Nom.
Two squares 2006-03-25
From Debbie:
A small square is constructed. Then a new square is made by increasing each side by 2 meters. The perimeter of the new square is 3 meters shorter than 5 times the length of one side of the original square. Find the dimension of the original square
Answered by Stephen La Rocque.
The kangaroo high school band 2006-03-12
From Cristy:
Two hundred students joined the kangaroo high school band. Based on statistics from previous years, 1/3 of the students will drop out. of the remaining members, only 6% will qualify for the state band. How many of the original members can be expected to qualify for the state band?
Answered by Stephen La Rocque.
How many students and how many cookies are there? 2006-03-10
From Stephanie:
A group of students is trying to evenly divide the cookies remaining from a bake sale. If each student takes three cookies, there are five cookies left. If each student takes four cookies, they are two cookies short. How many students and how many cookies are there?
Answered by Penny Nom.
25+36+12=52+m, m= 2006-03-02
From Jenny:
25+36+12=52+m, m=
Answered by Stephen La Rocque.
Separate 35 into two parts such that 4 times the larger is 4 less than five times the smaller 2006-03-01
From Barbara:
Separate 35 into two parts such that 4 times the larger is 4 less than five times the smaller
Answered by Stephen La Rocque.
8 - 5r / 6 = 3 2006-02-15
From Scott:
I was trying to help my daughter with the following problem:

8 - 5r / 6 = 3

Can you give me a quick refresher.


Answered by Stephen La Rocque.
How long and how wide is the table? 2006-02-02
From A student:
the perimeter of a table is 24 feet. the table is twice as long as it is wide. how long and how wide is the table?
Answered by Penny Nom.
The sum of two numbers is 21. 2006-01-31
From Dan:
The sum of two numbers is 21. One number is three less than the other. Find the numbers
Answered by Penny Nom.
The three angles in a triangle 2006-01-23
From A student:
the measure of the 2nd angle in a triangle is 4 more than the measure of the 1st angle. the measure of the 3rd angle is eight more than twice the measure of the 1st angle. find the measure of each angle.
Answered by Penny Nom.
y - y1 = m(x - x1) 2006-01-08
From Greg:
How do you change 2x + 3y = 7 into y - y1 = m(x-x1)?
Answered by Penny Nom.
Why do we bother learning pre-algebra and algebra 1? 2005-12-23
From Priya:
My students always ask "Why do we bother learning pre-algebra and algebra 1?" and I haven't found an answer to satisfy them yet. Can you help me? My students are from grade 9 to 11. I have tried giving them real life examples in each topic but it just feels like they are not satisfied!!!
Answered by Penny Nom.
A rectangle has a length that is 7cm more than twice the width 2005-12-21
From Stephanie:
A rectangle has a length that is 7cm more than twice the width. The area of the rectangle is 60cm squared. Find the dimensions of the rectangle.
Answered by Penny Nom.
Skeeball 2005-12-21
From Jenny:
Kira and Dajuan decided to have a Skeeball contest. They each played one game to see who would get the highest score. A description of their contest follows:
Answered by Denis Hanson.
If 7 times a number is decreased by 8 ... 2005-12-17
From Joyce:
If 7 times a number is decreased by 8, the result is the same as when 3 times the number is increased by 6. Find the number.
Answered by Penny Nom.
Pat invested a total of $3000 dollars 2005-12-14
From Duane:
Pat invested a total of $3000 dollars. part of the money yields 10 percent interest per year and the rest yields 8 percent interest per year if the total yearly interest is $256 how much did pat invest at 10 per cent and how much at 8 percent.
Answered by Penny Nom.
2/n +4 = 10- 4/3n 2005-12-11
From Jennifer:

2/n +4 = 10- 4/3n

5(x-3/4) = x+1


Answered by Penny Nom.
Solve for x 2005-12-04
From Lisa:
5(x-4)=3(2-3x)
Answered by Penny Nom.
An integer equation 2005-12-01
From Henry:
3x = 2y + 7

I know the answer: x=9 and y=10, but what are the algebraic steps to solve the equation?

Answered by Penny Nom.
Solve for g 2005-11-09
From Cassandra:
solve each equation or formula for the variable specified

5g+h=g, for g

Answered by Penny Nom.
Coefficients, constants and like terms 2005-10-05
From Elizabeth:
In the equation -8y+6ab+7-3ab what are the coefficients; the like terms and constants?
Answered by Penny.
2x/x^2-9 - 6/x^2+x-12 2005-09-27
From Emily Ann:
2x/x2-9 - 6/x2+x-12
Answered by Penny Nom.
6x+7=8x-13 2005-09-27
From A student:
show me how to do 6x+7=8x-13
Answered by Penny Nom.
14p - 8 = 22 + 20p 2005-09-27
From Tiffany:
14p - 8 = 22 + 20p
Answered by Penny Nom.
x = (y^2 - b^2)/2b 2005-09-04
From Bob:
while looking at x=(y2-b2)/2b on a table I was preparing for a graph, I saw that there is a point where x and y get closer to each other's value. By trial and error I tried to find the point where x=y. I saw a number which looked familiar and I realized it was the square root of 2. and so, if y=b(1+square root of 2) then x=b(1+square root of 2) also, or as x=y.
My question is: How can this result x=y when y=b(1+square root of 2) be algebraically derived from the equation x=(y2-b2)/2b without pre knowing it is so?

Answered by Penny Nom.
3A/4 +1=(2A-1)/3 2005-08-21
From Rachael:

I'm doing a practice test question and the answer I keep turning out is different from the answer on the answer sheet. I think I might be forgetting a step. The problem is:

3A/4 +1=(2A-1)/3

The answer is -16. If anyone could explain the steps to me to getting the answer I would be grateful. Whenever I do it I get something completely different.


Answered by Penny Nom.
Solve for x 2005-07-20
From Ed:
(x-1)/3 - 1 = (x + 2)/2
Answered by Penny.
Explain why 3(x+2) = 3x+2 is incorrect 2005-03-28
From Cynthia:
An algebra student incorrectly used the distributive property and wrote 3(x+2) = 3x+2. How would you explain to him the correct result, without the use of the distributive law?

Explain why the square of the sum of two numbers is different from the sum of the squares of two numbers.

Answered by Penny Nom.
A product equals a sum 2005-03-27
From Tor:
Can you please help me with a general formula for the following system:
2+2=2*2
3+3/2=3*3/2
4+4/3=4*4/3

and how will you prove the formula?


Answered by Penny Nom.
Multiplying two algebraic fractions 2004-10-31
From A parent:
Multiply:

x+2 x²-4
----- x -------
x-2 x²+x-2


Answered by Harley Weston.
The price of a book 2004-09-18
From Dorly:
If the number of toonies required to buy a book is nine more than the number of five-dollar bills required to buy the same book, determine the cost of the book.
Answered by Penny.
Pizza for Jack? 2004-09-16
From Grace:
Jack is playing pool with Jim for $1 a game. He has only $2 and decides to play until he goes broke or has $5, at which point he will quit and go out for a pizza with Jim(Dutch treat). Jack knows from past experience that he beats Jim 60% of the time. What is the probability that Jack will get to eat pizza? Hints: Let A be the 6x6 matrix defined by A=[aij], where aij is the probability that Jack will have $(i-1)after one game is he starts with $(j-1). For example, a23 - .40 since there is a 40% probability that Jack will end up with $1 after a game is he starts the game with $2 (If Jack wins 605 of the time, he must lose 40% of the time). Also, for example, a52 = 0 since there is no way jack can have $4 after one game if he had $1 at the beginning of the game. Since Jack will stop if he goes broke or accumulates $5, a11 and a66 are both 1.

Let x0 = [0 0 1 0 0 0 ] transposed, which we interpret as saying that initially Jack has $2 with a probability 1. Then Ax0 will represent the porbability of each amount of money, $0-$5, after one game. What is the probability that Jack will be able to eat pizza by computing Akx0 for large k and finding a limiting value.

Answered by Penny Nom.
Three word problems 2004-09-12
From Adrian:

1. Increasing area of a field: Julia's soybean field is 3m longer than it is wide. To increase her production, she plans to increase her to increase both the length and width by 2m. If the new field is 46m2(2nd power) larger than the old field, then what are the dimensions of the old field?

2. Width of a Football Field: If the perimeter of a football field in the NFL including the end zones is 1040 ft and the field is 120 yd long, then what is the width of the field in feet?

3. Fencing dog pens Clint is constructing two adjacent rectangular dog pens. Each pen will be three times as long as it is wide, and the pens will share a common long side. If Clint has 65 ft of fencing, what are the dimensions of each pen?


Answered by Penny Nom.
(2x+5y-6) + (3x-4y+12) 2004-09-04
From GG:
I am a grade 9 student and i do not understand a question. Plz help! OK here it is........ (2x+5y-6) + (3x-4y+12)
Answered by Penny Nom.
Simplify 2004-06-27
From Tiff:
Simplify:

-3x (x2 + 2)

(x + 2) (x + 3)

Answered by Penny Nom.
Volume of a box 2004-05-23
From A student:
The area of the top of a closed rectangular box is 252cm2, the area of the front of the box is 105cm2, and the surface of the box is 834in2 (or 5380.64cm2). What is the volume of the box?
Answered by Penny Nom.
Subtracting fractions 2004-05-11
From Filipe:

Question:
_5_ - __7__
6ab 8a


Answered by Penny Nom.
Area and perimeter 2004-03-12
From Sandy:
The rectange has an area of M square units and a perimeter of 2M units. What is the value of x?

The length of the rectange is 3, while the width of the rectange is x-1.

Answered by Penny Nom.
Three numbers are added in pairs 2004-02-12
From Jane:
When three numbers are added in pairs, the sums of the pairs are 22, 39, and 45. What are the three numbers?
Answered by Penny Nom.
Difference of squares 2003-11-24
From Susie:

Factor assuming that n is a positive #

Problem: (I will give it to you in words beacuse I don't know how to do exponents on the computer.) Forty-five r to the 2n power minus five s to the 4n power. I was hoping you could walk me through it not just give me the answer.


Answered by Penny Nom.
4(r-2) = r(2+8) 2003-11-20
From Angela:
I not going to give you the problem that I have, but a problem like it. 4(r-2) = r(2+8) I get mix up trying to solve a problem like this could you help me. So that I will be able to handle the ones on my homework.
Answered by Penny Nom.
Solve for Q 2003-11-03
From Anwaar:

The problem is:

QT - X
________ + H = HT
QS


Answered by Peny Nom.
Solve for x 2003-10-27
From Lonnie:
i need help with solving equations and formulas such as -3x+b=6x, for x. I do not understand how you find x.
Answered by Penny Nom.
Domain of a function 2003-10-14
From Karim:
Find the Domain of the Function
H(x) = Log3 (4x + 7)-10
H(x) = log base 3 times 4X plus 7 minus 10
i am having problem solving these kinds of problems.

Answered by Penny Nom.
5x - y 2003-10-11
From Candace:
math problem 5x-y for x=12 and y=14 do you mulitilpy 5x12 and than -the answer from 14 which gives you 46? is this right
Answered by Penny Nom.
Area of a rectangular room 2003-09-11
From Kelly:
The area of a rectangular room is 238 square feet. The width is 3 feet less than the length. What are the dimensions?
Answered by Penny Nom.
The quadratic formula 2003-08-31
From Alex:
I am using Houghton Mifflin's Precalculus with Limits book, 2nd edition. However, the first chapter encompasses Algebra review, and I am stuck on a problem. All that's required is to solve the following and verify using a calculator:
3y2+6y+2=0
I have solved the problem using the quadratic formula, but from what I remember, the quadratic formula is used in the case of equations following the AX2+BX+C=0 pattern. As the problem I am attempting uses a y-variable, can I still use the quadratic formula? Since I am not sure what route to take in solving this problem, I am hoping you can assist me.

Answered by Harley Weston.
The domain of 1/g(x) - 5 2003-07-03
From Barbara:
If the range of g(x) is ( neg. infinity,4] and the domain of g(x) is ( neg. infinity, infinity), how do I find the domain of 1/g(x) - 5?
Answered by Penny Nom.
ABCD*4 = DCBA 2003-05-07
From Dee:

In the following problem, if the letters A, B, C and D represent some number, what numbers would they represent so that:

ABCD*4 = DCBA


Answered by Penny Nom.
Two intersecting graphs 2003-04-23
From Patty:

a) graph the equation

x2 - y - 4 = 0

x2 + y2 = 9
on the same set of coordinate axes.

I did not have a problem with this.

The problem is part (b) of the question ask: Find all solutions of the system in part (a) algebraically. Express answers in decimal form, accurate to two decimal places.


Answered by Penny Nom.
The substitution method 2003-04-14
From Patty:

I need help with the following:

5x - 4y = 13

2y + 3x = -1


Answered by Penny Nom.
12*nC2 = n! 2003-03-30
From A tutor:
Solve for "N"

12*nC2=N!

Answered by Walter Whiteley.
Two algebra problems 2003-03-18
From Kayla:
1st question- If a+b divided by a equals 6 and b+c divided by c equals 9, compute the numerical value of a divided by c. ( Your answer will be mumbers only, no variables.)

2nd question- Three adjacent faces of a box (a rectangular prism) have areas of 7, 14, and 18 square inches. Find the volume of the box. (Hint: Choose variables to represent the three sides of the box and then figure out the problem.)

Answered by Peny Nom.
Monomials 2003-03-11
From Roxy:
Explain why (x+y)z is not equal to xz+yz? P.S. Z is an exponent
Answered by Penny Nom.
Sasha's candies 2003-02-18
From Pat:
Tick-tac-toe

Sasha and Trudy are playing tick-tac-toe. Sasha agrees to give trudy two pieces of candy for each game Sasha loses, and Trudy agrees to give Sasha three pieces of candy for each game that Trudy loses. After playing thirty games, Sasha has the amount of candy that he started with. If no ties occurred, how many games did Sasha win?

Answered by Paul Betts.
Factoring a trinomial 2003-01-26
From A student:
how do you factor: Xsquared + 8x + 12
Answered by Penny Nom.
Two numbers have a ratio of 5:3 2003-01-01
From Patricia:
Two numbers have a ratio of 5:3. The larger number is 36 more than half the difference of the two numbers. What are the two numbers?
Answered by Penny Nom.
Find a, b and c 2002-12-10
From Sameer:
Question:

a+b+c=180
b=a+(120/2-5)
c=b-(130-10)/6
a=?
b=?
c=?

Answered by Penny Nom.
[x*(x+2)] + 1 = (x+1)(x+1) 2002-11-27
From Melissa:
While teaching the multiplication table to my daughter I noticed an interesting pattern. It goes something like ...Take a whole number
...add two to it
...multiply the two numbers together
...add one to the resulting number
...it will now be equal to the original number plus one, squared.

Answered by Claude Tardif.
The substitution method 2002-11-15
From A student:
Solve by using the substitution method.

x = 3y - 1
x + 2y = 9


Answered by Penny Nom.
Algebra 2002-11-13
From Carol:
I am a confused parent trying to help my dau hgter solve some math problems and I need help.

3[x-4]=27


Answered by Penny Nom.
(93+n)+158=93+(28+158) 2002-11-10
From Parents:
(93+n)+158=93+(28+158)
Answered by Penny Nom.
G = 3H - 5 2002-11-07
From Kash:
G=3H-5, for H solve for the given variable
Answered by Penny Nom.
18m-11+ -7 2002-11-06
From Sophia:
what is 18m-11+ -7 (m = to 12)?
Answered by Penny Nom.
Mrs. Klein has been losing 1 pound a week for the last year 2002-10-30
From Brett:
On her diet, Mrs. Klein has been losing 1 pound a week for the last year. Her husband weighs 110 pounds more than she does now. If his weight is twice what her weight was 6 weeks ago, how much does mrs. Klein weigh now?
Answered by Penny Nom.
Jimmy's hits 2002-10-27
From Patty:
One seventh of Jimmy's hits were doubles
12.5% of his hits were homeruns
Jimmy had 82 singles
He did not have any triples

How many hits did Jimmy have? (Be sure to include the equation that represents the number of hits)

Answered by Penny Nom.
How many trees were planted in March? 2002-10-20
From Allison:
The number of trees planted by the Alpine Nursery in April was 3 more than twice the number of trees planted in March. If 71 trees were planted in april, write and solve an equation to find how many trees were planted in March.
Answered by Penny Nom.
Simplify 2002-10-19
From A student:
well i need to simplify this problems out and i cant do it so here is one
 3x^2+6x-45 ------------ = 3x^2+21x+30 

Answered by Penny Nom.
Three algebra problems 2002-10-13
From Veronica:
Solve the following inequality:

5(xsqured-4)/(xto the 5th(2x-5)to the 3rd) < or equal 0

Solve the following equations for all roots

square root of x+2=-1+square root of 2x+3

what's ZERO FACTOR PROPERTY???

The hypotenuse of an isosceles right triangle is 7cm. long. Determine the lenghts of the other sides.


Answered by Leeanne Boehm.
$100 is split between four guys. 2002-10-03
From Stephan:
$100 is split between four guys. A,B, C, and D. How much money does if person get if B has four more dollars than A, C has 8 more dollars than B, and D has twice as much money as C?
Answered by Penny Nom.
Formulating equtions 2002-09-14
From Theresa:
Hello. In math class we have to formulate equations for certin questions for extra credit. We are allowed to ask anyone and anyone can do it for us and explain it to us. Could you please do that? Here are the questions
  1. Jordan has 3 times as much money as Reva. Together they have $44. How much money does each have?

  2. The Tigers played 48 games. They won twice as many as they lost. How many did they win?

  3. There are 900 students in Sewickley Academy. There are 20 more girls than boys. How many girls are there?

  4. A board 400 cm long is cut into 2 pieces. One piece is 66 cm longer than the other. Find the lenght of the shorter piece.


Answered by Penny Nom.
Algebra 2002-09-08
From Heather:
This question is from an 8th grade Algebra classs. I am a parent asking for help to explain to my child. Q: d=25t2.
Answered by Leeanne Boehm.
Express 5120 as a sum of consecutive numbers 2002-08-25
From Todd:
Express 5120 as a sum of more than 1 consecutive number.
Answered by Leeanne Boehm.
Variable expressions 2002-08-15
From Brittany:
I am just learning variable expressions. My homework is to write a variable expression for each word phrase;

example :

the number of minutes in s seconds
the cost of x cartons of juice at $.75 each
And etc.

I don't understand it

Answered by Penny Nom.
x + 5 + x, if x = 4 2002-07-23
From Elisa:
x + 5 + x, if x = 4
Answered by Leeanne Boehm.
Jack and Jill wrote letters 2002-06-19
From Angel:
jack and jill each bought a stationary
# of sheets of paper in each box were the same
# of envelopes were the same
jill wrote letters consisting of 3 pages
jack wrote letters consisting of 1 page

when they wrote all letters jill had 50 envelopes left and jack had 50 sheets of paper left how many pieces of paper and how many envelopes?

Answered by Leeanne Boehm.
The cross country team 2002-06-12
From Denae:
In cross country, a team's score is the sum of the first five finishers on the team. The captain of the team finished 2nd in the meet. The next four finishers on the team placed in consecutive order. The team score was 40. in what places did the other members finish?
Answered by Penny Nom.
A polynomial 2002-06-05
From Melissa:
I'd like to know what is a polynomial( the definition and an explication)? And is 7x a polynomial? and why?
Answered by Penny Nom.
One-fourth of a number is added to one-third of the same number 2002-05-26
From A student:
When one-fourth of a number is added to one-third of the same number, the result is 28. What is the number?
Answered by Penny Nom.
| n + 4 | + | 3 - 2n | = 16 2002-05-24
From Randall:
I don't seem to be able to solve this: | n + 4 | + | 3 - 2n | = 16
Answered by Penny Nom.
Find the angle measures 2002-05-18
From Amanda:
In triangle ABC; the measure of angle A is 20 degrees more than twice the angle B. The measure of angle C Is five times angle B. Find the angle measures.
Answered by Penny Nom.
Some algebra 2002-05-11
From A student:

/x - 2 ^4/x = 8

/ = square root sign
^ = 4th root of x

also, dividing
2x3 -3 -6x
__________
4 + 2x

the 2x3 is 2x to the third


Answered by Penny Nom.
3 = -2x 2002-05-05
From Timothy:
My question is 3 = -2x

How do I isolate the variable here?


Answered by Penny Nom.
Factor 12x^2 - 20x - 8 2002-04-30
From Crystal:
Factor Completely: 12x2 - 20x - 8
Answered by Penny Nom.
25m = 100 2002-04-22
From Megan:
If 25m = 100, Then m equals what number?
Answered by Penny Nom.
An augmented matrix 2002-04-20
From A student:
Hi my math teacher asked us to solve an augmented matrix. I am in twelfth grade and need help. The book we are working on is college algebra. Here it is

a+2b+c=0
2a+5b+4c=-1
a-b-9c=-5


Answered by Penny Nom.
Solving for x 2002-04-19
From Susan:
I'm having a little trouble solving these equations for x. I can't seem to separate x fully from the other numbers. Please help! Here are the problems:

(2x-1)/(x-2)(x 2+3) = 0 and

y/(x+1)=z/x
Answered by Penny Nom.

Two algebra problems 2002-04-14
From Ashley:
Question 1: For all values of a, let < a > be defined as < a > = 4a - 4 . Which of the following equals < 6 > - < 5 >? A: < 2 >
B: < 3 >
C: < 4 >
D: < 5 >
E: < 6 >

Question 2: If 5n + p = 3 and 2m - 10n = 2, what is the value of m + p?


Answered by Penny Nom.
3/4n-6=12 2002-04-07
From Diane:
I am in 6th grade advanced math and I just started Algebra. My teacher tries to explain it to me but I just can't get it. I am having difficulty understanding it. Please HELP!!! I will type a problem, if you could answer it and show your work so i can understand it that

would be great. Please explain in simple terms.

here is a couple problems I am having trouble with,

PROBLEM #1

3/4n-6=12


Answered by Penny Nom.
If the matrix A is inverible and AB =AC, then B = C 2002-03-27
From Vikki:
i hope you can help i am soooo stuck here goes:

a)

 A= 0 1 B= 1 1 c= 2 5    0 2    3 4    3 4 
A,B and c are matrices

Evaluate AB and Ac (which I can do)

then

b)
I need to prove that if the matrix A is inverible and AB =AC, then B = C. Why does this not contradict what happened in part a)?
Answered by Leeanne Boehm.

Adding algebraic fractions 2002-03-20
From Dolores:
I get totally confused with this problem. I get confused with the getting the lowest terms.

4/x-5 + -2/x -10/x^2-5x


Answered by Penny Nom.
Simplify this expression 2002-02-26
From Francine:
2X power 5 Y power 3 times 18 X power -5 Y power -3 divided by X power 1/3 Y power 1/4.

I came up with
     36 ---------- X 1/3 Y 1/4 
The book says 36 x1/3 y1/4.
I don't get it.

Answered by Penny Nom.
Adding algebraic fractions 2002-01-23
From Francine:

Hello, I've come up with an answer for the following problem but it's not the same as the book's answer. However, the book has a lot of mistakes in it and I don't know if I'm doing it properly or not. Your help would be soooo appreciated. It's driving me crazy.The problem is:

   
  2             + 3          + 4 
---------  +  ---------  +  ---------- 
(x-1)^3        (x-1)^2       (x-1) 

[(x-1)^3 is (x-1) to power of 3 etc.]


Answered by Penny Nom.
Algebra review questions 2002-01-14
From A student:
  1. A wildebeast was observed racing a distance of 86.4m in 4.8 seconds. Find the speed of the wildebeast in m/s.

  2. 8x - 7 - 5x = 23

  3. 7 z - 3 (z + 2) = 26

Answered by Penny Nom.
A simplification problem 2002-01-10
From A Parent:
FIND

(A / B) . (C / D)
where
A = Y squared + 3 . Y cubed
B = Y squared + 4
C = 2 . Y + Y squared
D = Y + 4 Y squared + 3 Y cubed


Answered by Penny Nom.
Two algebra problems 2001-12-24
From Cass:
if the graph of a equation is 2x2 - y2 = 8 passes through point (6,k), find the positive value of k.

if 5x2 - 2 x = 1, find positive value of x


Answered by Penny Nom.
Composition of functions 2001-12-16
From Paula:
  1. if f(x)= 3x-1 and g(x)= 1/2x + 3 find fog(2)

  2. find the values of x for which tanx=0

Answered by Penny Nom.
Algabra 2001-12-13
From Stephanie:
My name is Stephanie. My grade 7 class is doing some strange form of algebra which I've never done nor seen before. We are doing equations like these: 3y - 6 + 7 - 4y = 6y - 6. I don't really understand them and was wondering if you could help me do my homework and explain how to at the same time. It would be really great if you could!!!!! Thank you so much! -Stephanie
  1. -9x - 5 - 8 + x = ?

  2. 5 + 6x - 3y + x + 8y = ?

  3. -x - 3 + 5x + 6y + 8x - 9= ?

Answered by Penny Nom.
y = zx - x - c 2001-12-11
From Andrew:
Could you please help me work out the value of x where: y = zx - x - c
Answered by Penny Nom.
y= (3x+2)/(4x-5) 2001-12-05
From Courtney:
y= (3x+2)/(4x-5)
solve for x

Answered by Penny Nom.
what number doubled plus 20 will give you 4/3 the original number 2001-11-15
From Hank:
what number doubled plus 20 will give you 4/3 the original number

2 x__ + 20 = 4/3 of the original number

Answered by Penny Nom.
(a+b) squared and a squared plus b squared 2001-11-14
From Kristen:

(a+b)2 = a2 + b2

Is this true for all whole numbers, and why?


Answered by Penny Nom.
A cubic 2001-11-10
From Louise:
x3 + 9x2 - 7x - 63
Answered by Penny Nom.
1 minus 5(y) equals negative 2 2001-11-10
From Judy:
1 minus 5(y) equals negative 2
Answered by Penny Nom.
A circle and triangle overlap 2001-11-09
From Tara:
A circle and triangle overlap as shown.the area of the circle is three times the area of the triangle.If the common region is removed,then the area of the rest of the circle would be 14 sq cm more than the area of the rest of the triangle.How many sq cm are in the area of the complete triangle.
Answered by Penny Nom.
8w-15 = 3w-10 2001-11-01
From A student:
8w-15=3w-10, w =?????
Answered by Paul Betts.
m - 8 2001-10-30
From Danita:
Can you please e-mail me the answer to m -8 for m = 20 and how you got the answer
Answered by Penny Nom.
An algebraic fraction 2001-10-25
From brandi:
4x - 5
_____ =4
3-7x

Answered by Penny Nom.
Dimensions of a frame 2001-10-16
From Rachel:
A rectangular picture frame has a perimiter of 44.2 cm.The width of the frame is seven tenths of its length what are the dimensions of the frame?
Answered by Penny Nom.
Some algebra 2001-10-15
From James:
I cannot figure these out I was wondering if you could help me? I have no one to answer my questions.
  1. (7x2 – 3yz)2 – (7x2 + 3yz)2

  2. Use Pascal’s triangle to expand (2x – y)4

  3. 8x3 y - x3 y4

  4. (m + 3n)2 – 144

  5. 12x4 y – 16x3 y2 – 60x2 y3

  6. p3 q2 – 9p3 + 27q2 – 243

Answered by Peny Nom.
Simplify the expression 2001-10-08
From Natalia:
simplify expression

4(x-7)+(-5x)=-10


Answered by Leeanne Boehm.
Investing in a yacht 2001-09-26
From Junior:
Marvin and his four friends wish to invest in a yacht by sharing equally in the cost. As the date of sale approaches, Marvin realizes that if three more people join the partnership, each partner's share would decrease by $18,000. How much does the yacht cost?
Answered by Penny Nom.
Solve for h 2001-09-17
From Kim:
s=2lw+2wh+2hl solve for h
Answered by Penny Nom.
Multiplying binomials 2001-09-08
From A student:
(x+1)(x+2)=?
(2x-1)(x-4)=0
please tell me this question!!

Answered by Leeanne Boehm.
Matrix reconstruction 2001-07-19
From Guy:
Is there a way to get the sums of rows, columns and diagonals of an n x n matrix to reconstruct the original matrix?
Answered by Walter Whiteley and Patrick Maidorn.
Simplifying algebraic expressions 2001-05-12
From A student:
How do you simplify algebra expressions?
Answered by Penny Nom.
Solving some equations 2001-04-16
From Justin:
Solve the following equations for x
  1. x+7=10
    x=

  2. x-7=10
    x=

  3. 3x=7
    x=

  4. x/3=10
    x=

  5. .

  6. .

  7. .

  8. .

Answered by Penny Nom.
Parabola problems 2001-04-10
From Kathleen:
  1. Graph each function and state its domain and range. y = 3x2 + 4

  2. For each parabola find: i) the direction of opening
    ii) the coordinates of the vertex
    iii) the y-intercept
    iv) the x-intercepts
    y = x2 + 3

  3. Find the equation of each parabola vertex at (0, -2) and passing through the point (3,7)

Answered by Harley Weston.
(8 + 5)5 = (y + 4)4 2001-04-01
From Kelley:
(8 + 5)5 = (y + 4)4
What is the value of y?

Answered by Penny Nom.
Factoring 2001-03-22
From Kaleena:
Factor this equation: 5x7-10x5+4x3-8x
Answered by Harley Weston.
Radicals 2001-03-18
From Nikki:
3) squareroot of x2 + 10x + 25

4) sixthroot of (m + 4)6

5) cuberoot of -64r6w15

.
.
.
.

Answered by Penny Nom.
Powers 2001-03-04
From A student:
Hey, can you show me how you do ..

(2xy)to the 3rd power (x) to the 2nd power?


Answered by Penny Nom.
What is the value of n? 2001-03-03
From Adam:
When I add n to 7 and then divide my answer by 4,I get 5. What is the value of n?
Answered by Penny Nom.
Solve for the variable 2001-02-28
From Ashley:
I've got 2 questions. We (my dad and I) can not figure them out. please help me.
  1. 21 = 9 - 2 (4a + 2) and I am supposed to find out what the variable is.

  2. 2/3 n + 3/8 n = 15/16 (those are fractions). Again, I need to find out what the variable is.

Answered by Penny Nom.
Three sheep 2001-02-28
From A student:
A farm sold three sheep. the weight of the three sheep combine wae 152 lbs. The smallest sheep weighted 7 lbs less than the middle size sheep. The largest sheep weighted three times the smallest sheep. what is the weight of each sheep?
Answered by Penny Nom.
Difference of Squares 2001-02-22
From Bruno-Pierre:
I noticed the other day that if you substract two consecutive squared positive numbers, you end up with the same result as if you add up the two numbers.

Ex. 5 and 6 (2 consecutive positive numbers)
52 = 25
62 = 36
36 - 25 = 11 (Substraction of the squared numbers)
5 + 6 = 11 (Sum of the numbers)

A more algebric view:
a2 - b2 = a + b where a and b are consecutive positive positive numbers. (b = a + 1)

I wondered if this rule had a name, and who discovered it.


Answered by Penny Nom.
Difference of squares 2001-02-20
From Janna:
Hi! I was just wondering how you would factor x2 - 9y2.
Answered by Harley Weston.
g(g+5)=0 2001-02-20
From Jenna:
g(g + 5) = 0
Answered by Penny Nom.
Dividing polynomials 2001-02-19
From Janna:
I have two questions involving dividing polynomials by polynomials.

Here's the first one:

Two factors of 12a4 -39a2 + 8a - 8a3 + 12 are a - 2 and 2a + 1. Find the other factors. The other question I'm stuck on is: When 10x3 + mx2 - x + 10 is divided by 5x-3, the quotient is 2x2 + nx - 2 and the remainder is 4. Find the values of m and n.


Answered by Penny Nom.
Adding functions 2001-02-19
From Jackie:
f(x) = x2 + 1; g(x) = x2 - 1

find
a) ((f+g) of h)) (x)
b) ((f-g) of h)) (x)
c) ((f of h) - (g of h)) (x)


Answered by Penny Nom.
A quartic equation 2001-02-15
From George:
Let P(x) = x4 + ax3 + bx2 + cx + d. The graph of y = P(x) is symmetric with respect to the y-axis, has a relative max. at (0,1) and has an absolute min. at (q, -3)

a) determine the values for a, b c, and d using these values, write an equation for P(x)
b) find all possible values for q.


Answered by Harley Weston.
Linear equations 2001-02-13
From Jamie:
I have a question how do you do linear equation?
Answered by Penny Nom.
Rewrite as an addition problem and Simplify 2001-02-13
From A student:
On my homework sheet it says "Rewrite as an addition problem and Simplify" and the gives this problem "16 - 120 ="
how do i do that problem?

Answered by Penny Nom.
L - R 2001-02-12
From A student:
If L = (a-b)-c and R=a- ( b-c), then L-R = ???
Answered by Penny Nom.
Chris' cards 2001-02-11
From A student:
Chris gave Jane x cards. He gave Bety one card more than he gave Jane and he gave Paul two cards fewer than he gave Betty. In terms of x, how many cards did Chris give Bety, Jane, and Paul together?
Answered by Penny Nom.
Some algebra 2001-01-23
From Thomas:
If a is 4 more than the quotient of b and 3, and if b=2, then what is the value of a?
Answered by Penny Nom.
A piecewise linear equation 2000-11-24
From Jacky:
There is a light bulb and it is given that the light bulb cost $0.75 and the cost of operating it is $0.0081 per hour. From the information give, I came up with the linear equation: Let c be total cost and Let h be hours used. Therefore: c = 0.0081h + 0.75 represents the total cost of the light bulb and the electricity. However, the second part of the question added the fact that the light bulb will only last for 800 hours. If the light bulb is replaced as soon as it burns out exactly after every 800 hours, how can I write an equation that represents that? Is it possible? What would it look like on the graph.
Answered by Harley Weston.
Integral solutions 2000-11-20
From David:
determine all pairs of integers (x,y) which satisfy the equation

6x2 - 3xy - 13x + 5y = -11

I got

y= (6x2-13x+11)/(3x-5)

and i don't konw how to continue


Answered by Claude Tardif.
Solving an equation 2000-11-14
From Heather:
4x-3 = 3x+4 these kinds of problems i dont get but everything else in my math class i have got. i just dont understand it.
Answered by Penny Nom.
Square roots 2000-10-29
From Pamela:
HERE GOES(I WILL USE Q AS THE SYMBOL FOR SQUARE ROOT):

8(Q2) - 5(Q2) + Q2

SECOND PROBELM IS

(1 + Q2)2

LAST ONE

3/(2-Q5)


Answered by Claude Tardif.
Mr. Edgecomb's math exam 2000-09-17
From Rachel:
In Mr. Edgecomb's math class, 30 students took an exam on statistics. If the average passing grade was 84, the average failing grade was 60, and the overall average was 80, how many students passed the test?
Answered by Penny Nom.
Trinomial 2000-09-06
From Lea:
I need to find the trinomial value of k which makes a perfect square. The problem is x2 - 6x + k
Answered by Penny Nom.
A equation with a square root 2000-09-03
From Lea:
Please help with 2 plus the square root ( and under it has x plus 1) = 6
Thank you

Answered by Penny Nom.
Subtracting fractions 2000-08-26
From Sarah Stanczyk:
how is this problem solved??

3/ x+1 - 5/x
( 3 divided by x +1 subtracted by 5 divided by x)


Answered by Penny Nom.
Two linear systems 2000-08-01
From A student:
please help me with this homework question. 5/x + 6/y= 19/6 3/x + 4/y =2
hint: let a = 1/x and b = 1/y substitute these expressions into the system to find a and b. Then find x and y.

This is a question I answered wrong on a test: solve the given system for x, y, and z. Express your solutions in terms of a, b, and c.


Answered by Penny Nom.
LOG(LN(x)) = 1 2000-07-28
From An algebra student:
LET F(x)=LOG X AND G(x)= LN X. SOLVE (f *G)(x)= 1
SHOW COMPOSITION AND USE DEFINITION OF LOGS.

Answered by Harley Weston.
Completing a square 2000-07-10
From Lexa Michaels:
How do you complete a square? the textbook question is: x 4 + 64
Answered by Penny Nom.
Simplify 21+2[3z+5(3z+8)] 2000-07-02
From Dennis Dyer:
My daughter has a math problem that I can't help her with. The problem is 21+2[3z+5(3z+8)] I can get the answer 101 +36z but I can't show her the correct way to write it out. Would you please show the correct way to show her work...
Answered by Penny Nom.
Simplifing 2000-06-15
From Angie Herbert:
My daughter is having problems with her algebra homework. She is a year 7 pupil at high school in the UK. She has been given homework on simplifying formulae and she doesn't understand how to do them. Here is an example of one of the questions :

9s+7t+5c+5s-3t

can you help her to understand how to do these and perhaps me too.
Answered by Penny Nom and Claude Tardif.

Rectangles and algebra 2000-06-13
From Kirstin:
A rectangle's length is 4 more than twice its width. The area of the rectangle is 336m squared. What is its length?
Answered by Paul Betts and Penny Nom.
Two problems 2000-06-12
From Sharon:
If f(4)=0 and f(6)=6, which of the following could represent f (x)?

A. 2/3x-4 B. x+2 C. x-4 D. 3/2x+6 E. 3x-12

these are problems to study for a test so I need to know the answer and how it was solved!

I have one more question

If 180o < theta < 270o and tan theta = 4/3, then sin theta =?

A. 5/4 B. -4/5 C. 3/5 D. 12/5 E. -3/5


Answered by Harley Weston.
A conic in standard form 2000-05-18
From Tara McConkey:
Im havign trouble converting the following conic to standard form, i know that the conic is a hyperbola but that is all 16x2-9y2-160x-18y+247=0
Answered by Harley Weston.
Subtracting polynomials 2000-05-13
From Teresa:
I am suppose to subtract the lower polynomial from the one above it.
-3a+11b+2f
-3a- 6b+8f
________________

Answered by Penny Nom.
4x - [2x -(3x - 2)] = 13 2000-05-13
From Teresa:
I need to find the following equation.
4x - [2x -(3x - 2)] = 13

Answered by Penny Nom.
Trig functions 2000-05-09
From Melissa:
Find all solutions in the interval (0,2pi)
2cos2x-3cosx-4=0

Answered by Paul Betts and Harley Weston.
Uniting algebra and geometry 2000-04-16
From Beth:
Who is the mathmatician that united algebra and geometry???
Answered by Claude Tardif.
Finding a Quadratic Equation given the roots 2000-04-16
From A secondary student:
How do I find the equation for the quadratic with roots at 2-i and its complex conjugate?
Answered by Harley Weston.
Increasing the weight 2000-03-23
From Lee Millard:
A weight of dimension a, b is to have its weight increased by w% by bolting plates of the same material along the top and one side (but not at both ends). The top plate and side plate have the thickness, x. Find the thickness of the plates for a given weight increase of w%.
Answered by Penny Nom.
A system of equations in five unknowns 2000-03-20
From Will:
I have been having some problem with the following question for some time. I would appreciate any help on solving the problem or a solution.

Q: Assume that a system of equations in the unknowns x1, x2, x3, x4 and x5 when converted to row echelon form gives

.
.
.

Answered by Penny Nom.
Simultaneous equations 2000-03-11
From Laura Molck:
My name is Laura Molck and I am in Year 11 in Australia. Please help me with the following. I know that they are all simultaneous equations which I can do but I have trouble with the formulae to work the equations. Can you please help!!

1. A tent manufacturer produces 2 models, Outback and Bushwalker. From earlier sales records it is known that 20% more of the Outback model is sold than the Bushwalker. A profit of $200 is made on each Outback sold, but $350 is made on each Bushwalker. If during the next year a profit of $177,000 is planned how many of each model must be sold?


Answered by Penny Nom.
Some equations 2000-03-08
From Maria:
Hi, could you help me with these questions:

Find x & solve

  • 7-2x/3=9
  • 4x+3/5=-2
  • 6-2x=14
  • -1/7x=-4/3+1/6

Answered by Penny Nom.
Fractions in algebra 2000-02-27
From Leslie:
Question:
add (4x+1)/(x-8) + (3x+2)/(x+4) + (49x+4)/(x^2-4x-32) and

solve x - 6/(x-3) = 2x(x-3)

Answered by Penny Nom.
Irrational algebraic functions 2000-02-21
From Bucky Cadena:
Here is the multipart problem:

Given f(x) = x-3* squareroot of x + 4

What does the f(x) intercept equal Find the two values for which f(x) = -5 Find the one value for which f(x) = -3


Answered by Harley Weston.
Can't make an equation! 2000-01-30
From Maggie Clarke:
An estate is to be distributed among the wife, 3 children and 2 grandchildren. The children will each receive 2x as much as each grandchild, and the wife will receive 4x as much as each child. If the estate amounted to $115,000, how much will each person receive?
Answered by Penny Nom.
Quadratics 2000-01-27
From Lori:

Hi my name is Lori. I'm a student, in the 11th grade.I have a problem in quadratic functions. I don't know how to get the numbers into the problem and then how to do the problem. here is one problem the rest are sort of similar to it... y=2x squared. If I were to find out how to get the (x,y) answers then I could graph it but as I said I can't find the numbers to go into the problem.

Also in the equation g: x arrow to y = -x + 4..... how would you know weather it is or isn't a quadratic equation????


Answered by Penny Nom.
Quadratic 2000-01-26
From Karen Johnson:
What is the reasoning behind the title quadratic equation?
Answered by Penny Nom.
Complex Roots 2000-01-24
From Jess Rutherford:
How do I find the value of k when 5x2 + k = 3x and has complex roots ?
Answered by Penny Nom.
Word problem 2000-01-24
From April:

Ok, my algerbra teacher gave me this problem and by just looking at it I can tell what the answer is but he wants me to show how I got the answer in an equation form.

The tempature at 7am on a cold day was -4 degrees Celcius. At 3pm the tempature was 1 degree Celcius. Write and solve an equation to find out how many degrees the tempature rose between 7am and and 3pm.


Answered by Penny Nom.
Domain of a function 2000-01-19
From Stacey Hutchison:

What is the domain of:

f(x) = -1/(x + 4)


Answered by Patrick Maidorn.
Algebra 2000-01-18
From Candis:
I want to know what exactly is Algebra?
Not how to do it, or methods. Just what is it?

Answered by Allen Herman.
Ax + By = C 2000-01-18
From Fatiha Khanboubi:
I was wondering if you could please explain to me how I would write the equation of line through the given points [(4,1), (6,3)] in Ax+By=C form.
Answered by Harley Weston.
Solving an equation 2000-01-17
From Shelley's Mom:

My daughter has an assignment in math that includes this equation:

-6b+5= -10

I've tried to help her with a scale. We've figured that you subtract 5 from each equation so its not -6b = -15. Is this correct and can you help us figure out the next step?


Answered by Penny Nom.
Algebra 2000-01-13
From Tim Merrill:
"please help me out with these four algebra questions"

  1. Rewrite the following using positive exponents only.

    xy -3
    -------
    x -2

  2. Find the following quotients.

    9a3+3a2+6a
    --------------
    3a


.
.
.

Answered by Penny Nom.
Parallel lines 2000-01-12
From Lori:
Find an equation of the line parallel to the given line containing the given point. x + y = 2; (1,2) We know parallel line have the same slope and we know the answer y = -x + 3.
Answered by Penny Nom.
Three algebra problems 1999-12-28
From Stephanie Branton:
  1. If P represents the product of all prime numbers less than 1000, what is the value of the unit's digit of P?

  2. Do any real numbers a and b exist such that: ln(a+b)=ln a + ln b? if so, what are they?

  3. Define a function by: f(x)=1/1-x where x is not equal to 0,1. what is f(f(f(a)))?

Answered by Harley Weston.
Two algebra problems 1999-12-17
From Michael Standfest:
If x+4 is a factor of 2x4+kx3-3kx2+6x-40, find k

and

Prove that n2-n is even for all n, using the proof of contradiction
Answered by Penny Nom.

Solve for v 1999-12-09
From Beth:
1-3/4(v+2)=-5
Answered by Penny Nom.
Two conics 1999-12-07
From Quinn:
I know the formula to find the axis of symmetry of a conic section (I'm not sure what shape - circle for the first one??) is (-D/2A,-E/2C) but I obviously don't get how to calculate it, because when I check the answer it's wrong, but I'm so close!! For the following equations my teacher suggested to "divide the x term coefficient, D, by the x squared term coefficient before...do the same for y."

2x2+2y2-8x+12y+16=0
.
.
.
Answered by Penny Nom.

The sixth game 1999-11-16
From Stephanie Sprunger:
Janice scored 18,21,27,16,and 19 points in five basketball games. How many points must she score in the next game to raise her average to 22 points per game?
Answered by Penny Nom.
Multiple step problems 1999-11-09
From Maria Rainsdon:
I don't understand how to solve these multiple step problems. I wasn't there when my teacher explained it and I am really lost. Here is one of the probs I don't understand: 3m+6=-m-6.
Answered by Penny Nom.
Transforming a triangle 1999-11-02
From Wilder Maldonado:
The height, H, of a triangle is increased by a lenth M, How much must be subtracted from the corresponding base,b, so that the area of the new triangle is one-half that of the original triangle.
Answered by Penny Nom.
Equations with fractions 1999-10-10
From Lori:
I'm a secondary student. How would you figure out a problem such as t-2/t =14/3t - 1/3. The book gave one example but it is really easy. The work YOU have to do is totally different. You don't have to tell me the answer just how to do such problems like these.
Answered by Penny Nom.
Common factors 1999-10-06
From Tommygirl98:
Ok I have a question about common factoring I'm not to sure if you could help me but it's worth a try here it goes. Ok I have a test on Friday and i'm having trouble understanding how to do it here's an example,

Factor,
-10 + 2b how do you do it and what do they mean by factor?
Answered by Penny Nom.

-8*-(-5a)=15 1999-09-22
From Shelby:
-8*-(-5a)=15

i cannot get this question!! please help me!!
Answered by Harley Weston.

Bob and his dad 1999-09-14
From Yvette:
"Bobs dad is 27 yrs older then Bob. The sum of their ages 5 yrs ago was 45 yrs. How old is Bob now?"
Answered by Penny Nom.
The sum of the cubes is the square of the sum 1999-08-25
From Bernard Yuen:
How to prove 13 + 23 + 33 + 43 + ... n3 is equal to (1+2+3+...n)2? (for n is positive integer)
Answered by Harley Weston.
The three cousins 1999-06-01
From Irena:
I have three cousins on my dad's side. The product of their ages is 84. Next year the youngest will be 1/4 the age of the middle cousin. In 2 years from now, the oldest will be twice the current age of the middle. What is the age of my oldest cousin?
Answered by Harley Weston.
A problem with powers 1999-06-01
From Bill:
solve for the variable:

5-x = 25x+6
Answered by Penny Nom.

Graphing a linear equation 1999-05-27
From Dylan Bradley:
Im in Grade ten and am doing linear equations, I cann't figure out how to make a chart for questions like y = -2x + 3
Answered by Penny Nom.
Introductory Algebra 1999-05-19
From Pat:
(2 + sq. root of 3) x (2 - sq. root of 3) = 1

Please show me the work.
Answered by Harley Weston.

A 1999-05-02
From Leah:
a=b
a^2=ab
a^2+b^2=ab-b^2
(a-b)(a+b)=b(a-b)
a+b=b
b
2=1

why is this proof wrong?
Answered by Penny Nom.

y=mx+b 1999-04-15
From Selena Royle:
When solving a problem on slopes,and in the formula y=mx+b what do the M and B stand for? Why M and B?
Answered by Penny Nom.
Extra point questions 1999-04-15
From W Troy Council:
My teacher has given me 5 extra point questions. I have figured out two, but i need a little assistance with 3 others, could you help me with:
  1. A stablehand agreed to work for 1 year. At the end of that time, she was to receive $240 and one horse. After 7 months she quit the job, but still received the horse and $100. What was the value of the horse?

  2. If the roots of x2 + bx + c are pi and square root 2, then find "b" and "c."

  3. ...

Answered by Penny Nom.
Grade 10 math 1999-04-08
From Ron:
the type of question i'm having trouble with is 5(2x-1)=3(x-4)
Answered by Penny Nom.
Factoring 1999-03-08
From L. Sivad:
Question:
m2+6m+9-n2

Answered by Penny Nom.
An airplane problem 1999-03-08
From B.M.R.:
A plane left New York and headed East to its destination 3600 miles away across the Atlantic. On the way back its speed was boosted by a 50 mph tail wind and it arrived an hour early. What was its normal speed?
Answered by Jack LeSage.
Enlarging a Rink 1999-03-06
From Jennifer Rudd:
I've been having difficulty with this one question involving area. The level of the question is Grade 11/12. Here it is!
A rink is 40 m long by 20 m wide. There are plans to enlarge it by 700m2 by adding a strip at one end and a strip of the same width along one side. Find the width of the strip. (Let the width of the strip by x meters.)

Answered by Jack LeSage.
Algebra 1998-11-25
From Casey:
yeah my son brought home a math worksheet and he was wondering and me too if u could help him and me out . he has problems like this 4n+9=7n+2= ??? and he has to show work so if u could help us out that would be great
Answered by Jack LeSage.
Adding Fractions 1998-10-03
From Pam Bailey:
Can you help me simplfy this?

(1/2a + 1/3b) - (1/4a - 1/5b) + (1/6a - 1/7)

thanx
Answered by Harley Weston.

Grade 9 Algebra 1998-06-07
From Tarah Kostenko:
I am a grade 9 student and I cannot figure out how to solve these two similar problems. can you please help?
-3(y-1/2)=1/2 also -2/3(x + 1)=6
I don't come up with the same answers as the book and I can't figure out what I'm doing wrong.

Thank you,

Answered by Harley Weston.

Espaces vectoriels 2002-08-18
From Bacar:
Comment montrer que trois (ou plus que trois) sous espaces vectoriels sont en somme directe.
Answered by Claude Tardif.
Les equations 1999-12-09
From Enciso Alberto:
j'ai les equations suivantes:

X1=Rs + A1/(B+W^2)
X2=Rs + A2/(B+W^2)
X3=Rd + A3/(B+W^2)
X4=Rd + A4/(B+W^2)
X5=Rg + A5/(B+W^2)
X6=Rg + A6/(B+W^2)

dont les coefficients Ai (i=1,2,..) et B sont constants (mais on ne conais pas leur valeur) et Rs, Rd, et Rg sont inconnues. Nous pouvons constater que la dependence de ces differentes expessions en fonction de W^2 est identique. Le tracé de l'une de ces expressions en fonction d'une autre est donc une driote.
.
.
.
Answered by Claude Tardif.

 
Page
1/1

 

 


Math Central is supported by the University of Regina and The Pacific Institute for the Mathematical Sciences.

CMS
.

 

Home Resource Room Home Resource Room Quandaries and Queries Mathematics with a Human Face About Math Central Problem of the Month Math Beyond School Outreach Activities Teacher's Bulletin Board Canadian Mathematical Society University of Regina PIMS